Sei sulla pagina 1di 35

DAMM NOTES – LAW ON INTELLECTUAL PROPERTY

SY $%&! - !"#!
Atty. Castillo-Taleon


References: special appearance to and can serve as pattern for an industrial product or handicraft (IPC,
Intellectual Property Code of the Philippines (Republic Act No. 8293) Sec. 112).
WTO Agreement on Trade-Related Aspects of Intellectual Property Rights (TRIPS Agreement)
World Intellectual Property Organization (WIPO) Intellectual Property Handbook (2004) 4. Trademarks and Service Marks
TRADEMARK/ SERVICE MARK is any visible sign capable of distinguishing the goods
(trademark) or services (service mark) of an enterprise and shall include a stamped or
INTELLECTUAL PROPERTY RIGHTS IN GENERAL marked container of goods (IPC, Sec. 121.1)

5. Patents
I. INTELLECTUAL PROPERTY RIGHTS PATENTABLE INVENTION refers to any technical solution of a problem in any field of
human activity which is new, involves an inventive step and is industrially applicable (IPC,
Sec. 21).
Section 4. Definitions.-
4.1. The term "intellectual property rights" consists of:
6. Layout designs (Topographies) of Integrated Circuits
(a) Copyright and Related Rights;
INTEGRATED CIRCUIT is a product, in its final form, or an intermediate form, in which the
(b) Trademarks and Service Marks;
elements, at least one of which is an active element and some or all of the interconnections
(c) Geographic Indications;
are integrally formed in and/or on a piece of material, and which is intended to perform an
(d) Industrial Designs;
electronic function (IPC, Sec. 112.2).
(e) Patents;
(f) Layout-Designs (Topographies) of Integrated Circuits; and
LAYOUT DESIGN (TOPOGRAPHY) refers to the three-dimensional disposition, however
(g) Protection of Undisclosed Information (n)
expressed, of the elements, at least one of which is an active element, and of some or all of
the interconnections of an integrated circuit, or such a three-dimensional disposition
prepared for an integrated circuit intended for manufacture (IPC, Sec. 112.3).

INTELLECTUAL PROPERTY refers to creations of the mind: inventions, literary and artistic 7. Geographic indications
works, symbols, names, images, and designs used in commerce. GEOGRAPHIC INDICATION identifies a good as originating in the territory of a Member, or
a region or locality in that territory, where a given quality, reputation or other characteristic of
It means the legal rights which result from intellectual activity in the industrial, scientific, literary the good is essentially attributable to its geographical origin (TRIPS Agreement, Art. 22).
and artistic fields.

II. DIFFERENCES BETWEEN COPYRIGHT, TRADEMARK AND PATENT


COPYRIGHT TRADEMARK PATENT
Intellectual Property Rights confined to literary or artistic any visible sign capable of right granted to an inventor by
1. Copyright and Related or neighboring rights works which are original distinguishing the goods of the State, or by the regional
COPYRIGHT is confined to literary or artistic works which are original creations in the literary creations in the literary or an enterprise and shall office acting for several
or artistic domain protected from the moment of their creation (IPC, Sec. 172). artistic domain protected from include a stamped or marked States, which allows the
the moment of their creation container of goods inventor to exclude anyone
2. Protection of Undisclosed information else from commercially
UNDISCLOSED INFORMATION refers to information which: exploiting his invention for a
(a) Is secret in the sense that it is not, as a body or in the precise configuration and limited period
assembly of its components, generally known among or readily accessible to persons
within the circles that normally deal with the kind of information in question;
(b) Has commercial value because it is secret; and
(c) Has been subject to reasonable steps under the circumstances, by the person lawfully Right of a Foreigner to Sue for Protection of Intellectual Property Rights
in control of the information, to keep it secret (TRIPS Agreement, Art. 39).

3. Industrial designs Section 3. International Conventions and Reciprocity. - Any person who is a national or who
INDUSTRIAL DESIGN refers to any composition of lines or colors or any three-dimensional is domiciled or has a real and effective industrial establishment in a country which is a party to
form, whether or not associated with lines or colors. Such composition or form gives a any convention, treaty or agreement relating to intellectual property rights or the repression of
unfair competition, to which the Philippines is also a party, or extends reciprocal rights to

! DomingoŸAriolaŸMartinezŸMolaer

DAMM NOTES – LAW ON INTELLECTUAL PROPERTY SY $%&! - !"#!
Atty. Castillo-Taleon


nationals of the Philippines by law, shall be entitled to benefits to the extent necessary to give Whatever favor, allowance, consideration, privilege or immunity a member-state grants the
effect to any provision of such convention, treaty or reciprocal law, in addition to the rights to nationals of another country is “immediately and unconditionally” accorded to the nationals of
which any owner of an intellectual property right is otherwise entitled by this Act. (n) other member-states (TRIPS, Art. 4).

Section 160. Right of Foreign Corporation to Sue in Trademark or Service Mark


Enforcement Action. - Any foreign national or juridical person who meets the requirements of
Section 3 of this Act and does not engage in business in the Philippines may bring a civil or
administrative action hereunder for opposition, cancellation, infringement, unfair competition, or III. TECHNOLOGY TRANSFER ARRANGEMENTS
false designation of origin and false description, whether or not it is licensed to do business in
the Philippines under existing laws. (Sec. 21-A, R. A. No. 166a) A. Voluntary License Contract

Section 85. Voluntary License Contract. - To encourage the transfer and dissemination of
• Principle of Reciprocity technology, prevent or control practices and conditions that may in particular cases constitute an
The following are entitled to the benefits of the IPC: abuse of intellectual property rights having an adverse effect on competition and trade, all
Any person who is a national, or who is domiciled, or has a real and effective industrial technology transfer arrangements shall comply with the provisions of this Chapter. (n)
establishment in a country which:
1. Is a party to any convention, treaty or agreement relating to intellectual property rights Section 90. Rights of Licensee. - The licensee shall be entitled to exploit the subject matter of
or the repression of unfair competition, to which the Philippines is also a party; or the technology transfer arrangement during the whole term of the technology transfer
2. Extends reciprocal rights to nationals of the Philippines by law. arrangement. (Sec. 33-C (1), R. A. 165a)

If a foreign corporation not doing business in the Philippines is suing as a party of a treaty to Section 91. Exceptional Cases. - In exceptional or meritorious cases where substantial
which the Philippines is a signatory, the fact that it is suing under Section 3 of RA 8293 need benefits will accrue to the economy, such as high technology content, increase in foreign
not be alleged as the court must take judicial notice of such fact as it is embodied in and exchange earnings, employment generation, regional dispersal of industries and/or substitution
supplied by the Paris Convention which forms part of the law of the land, provided that the with or use of local raw materials, or in the case of Board of Investments, registered companies
party suing substantially complied with the requirements of the law (Puma vs Dassler, GR with pioneer status, exemption from any of the above requirements may be allowed by the
No. 75067). Documentation, Information and Technology Transfer Bureau after evaluation thereof on a case
by case basis. (n)
However, if a foreign corporation is suing under any other agreement other than RA 8293,
failure to allege reciprocity is fatal to foreign corporation’s cause, it being shown that it failed
to comply with the requirements of the law (Leviton Industries vs Salvador, GR No. L- VOLUNTARY LICENSING refers to the grant by the patent owner to a third person of a right to
40163). exploit a patented invention.

A VOLUNTARY LICENSE CONTRACT need not be registered in the IPO to be valid. However,
• Principle of Reverse Reciprocity it must be registered to be binding.
Any condition, restriction, limitation, diminution, requirement, penalty or any similar burden
imposed by the law of a foreign country on a Philippine national seeking protection of
intellectual property rights in that country, shall reciprocally be enforceable upon nationals of
said country, within Philippine jurisdiction (IPC, Sec. 231). • Prohibited Clauses (IPC, Sec. 87)
The following provisions shall be deemed prima facie to have an adverse on competition and
trade:
(1) Those which impose upon the licensee the obligation to acquire from a specific source
• National Treatment Principle capital goods, intermediate products, raw materials, and other technologies, or of
The Philippines, upon becoming a member of the World Trade Organization (WTO), has permanently employing personnel indicated by the licensor;
adhered to the TRIPS, which provides that protection afforded to the member-state (with (2) Those pursuant to which the licensor reserves the right to fix the sale or resale prices
respect to intellectual property) must be extended to the nationals of other member-states. of the products manufactured on the basis of the license;
(3) Those that contain restrictions regarding the volume and structure of production;
(4) Those that prohibit the use of competitive technologies in a non-exclusive technology
• Most-Favored Nation Principle transfer agreement;

! DomingoŸAriolaŸMartinezŸMolaer

DAMM NOTES – LAW ON INTELLECTUAL PROPERTY SY $%&! - !"#!
Atty. Castillo-Taleon


(5) Those that establish a full or partial purchase option in favor of the licensor; B. Compulsory Licensing
(6) Those that obligate the licensee to transfer for free to the licensor the inventions or
improvements that may be obtained through the use of the licensed technology; COMPULSORY LICENSING refers to the grant by the Director of Legal Affairs of a license to
(7) Those that require payment of royalties to the owners of patents for patents which are exploit a patented invention without the permission of the patent holder, either by manufacture or
not used; through parallel importation (IPC, as amended by RA 9502, Sec. 4).
(8) Those that prohibit the licensee to export the licensed product unless justified for the
protection of the legitimate interest of the licensor such as exports to countries where A compulsory license should be granted to any person shown capable to exploit an invention.
exclusive licenses to manufacture and/or distribute the licensed product(s) have
already been granted; • Grounds
(9) Those which restrict the use of the technology supplied after the expiration of the (1) National emergency or other circumstances of extreme emergency
technology transfer arrangement, except in cases of early termination of the (2) When public interest requires
technology transfer arrangement due to reason(s) attributable to the licensee; (3) Manner of exploitation of patent is anti-competitive
(10) Those which require payments for patents and other industrial property rights after (4) Public non-commercial use of the patent without satisfactory reason
their expiration, termination arrangement; (5) Patented invention is not being worked in the Philippines on a commercial scale although
(11) Those which require that the technology recipient shall not contest the validity of any capable of being worked
of the patents of the technology supplier; (6) Where the demand for the patented drugs and medicines is not being met to an
(12) Those which restrict the research and development activities of the licensee designed adequate extent and on reasonable terms
to absorb and adapt the transferred technology to local conditions or to initiate
research and development programs in connection with new products, processes or
equipment;
(13) Those which prevent the licensee from adapting the imported technology to local Section 93. Grounds for Compulsory Licensing. - The Director of Legal Affairs may grant
conditions, or introducing innovation to it, as long as it does not impair the quality a license to exploit a patented invention, even without the agreement of the patent owner, in
standards prescribed by the licensor; favor of any person who has shown his capability to exploit the invention, under any of the
(14) Those which exempt the licensor for liability for non-fulfillment of his responsibilities following circumstances:
under the technology transfer arrangement and/or liability arising from third party suits
brought about by the use of the licensed product or the licensed technology; and 93.1. National emergency or other circumstances of extreme urgency;
(15) Other clauses with equivalent effects.
93.2. Where the public interest, in particular, national security, nutrition, health or the
development of other vital sectors of the national economy as determined by the appropriate
• Mandatory Provisions (IPC, Sec. 88) agency of the Government, so requires; or
(1) That the laws of the Philippines shall govern the interpretation of the same and in the
event of litigation, the venue shall be the proper court in the place where the licensee 93.3. Where a judicial or administrative body has determined that the manner of exploitation
has its principal office; by the owner of the patent or his licensee is anti-competitive; or
(2) Continued access to improvements in techniques and processes related to the
technology shall be made available during the period of the technology transfer 93.4. In case of public non-commercial use of the patent by the patentee, without
arrangement; satisfactory reason;
(3) In the event the technology transfer arrangement shall provide for arbitration, the
Procedure of Arbitration of the Arbitration Law of the Philippines or the Arbitration Rules 93.5. If the patented invention is not being worked in the Philippines on a commercial scale,
of the United Nations Commission on International Trade Law (UNCITRAL) or the although capable of being worked, without satisfactory reason: Provided, That the
Rules of Conciliation and Arbitration of the International Chamber of Commerce (ICC) importation of the patented article shall constitute working or using the patent. (Secs. 34, 34-
shall apply and the venue of arbitration shall be the Philippines or any neutral country; A, and 34-B, R. A. No. 165a)
and
(4) The Philippine taxes on all payments relating to the technology transfer arrangement
shall be borne by the licensor. Section 96. Compulsory Licensing of Patents Involving Semi-Conductor Technology. -
In the case of compulsory licensing of patents involving semi-conductor technology, the
license may only be granted in case of public non-commercial use or to remedy a practice
determined after judicial or administrative process to be anti-competitive. (n)


! DomingoŸAriolaŸMartinezŸMolaer

DAMM NOTES – LAW ON INTELLECTUAL PROPERTY SY $%&! - !"#!
Atty. Castillo-Taleon


Section 97. Compulsory License Based on Interdependence of Patents. - If the 9502
invention protected by a patent, hereafter referred to as the "second patent," within the
country cannot be worked without infringing another patent, hereafter referred to as the "first
patent," granted on a prior application or benefiting from an earlier priority, a compulsory Period to File a Petition
license may be granted to the owner of the second patent to the extent necessary for the
working of his invention, subject to the following conditions: Section 94. Period for Filing a Petition for a Compulsory License. -
94.1. A compulsory license may not be applied for on the ground stated in Subsection 93.5
97.1. The invention claimed in the second patent involves an important technical advance of before the expiration of a period of four (4) years from the date of filing of the application or three
considerable economic significance in relation to the first patent; (3) years from the date of the patent whichever period expires last.
97.2. The owner of the first patent shall be entitled to a cross-license on reasonable terms to
use the invention claimed in the second patent; 94.2. A compulsory license which is applied for on any of the grounds stated in Subsections
93.2, 93.3, and 93.4 and Section 97 may be applied for at any time after the grant of the patent.
97.3. The use authorized in respect of the first patent shall be non-assignable except with (Sec. 34[1], R. A. No. 165)
the assignment of the second patent; and

97.4. The terms and conditions of Sections 95, 96 and 98 to 100 of this Act. (Sec. 34-C, R.
A. No. 165a)
Ground Period
If the patented version is not being worked in Before the expiration of a period of four (4)
the Philippines on a commercial scale, years from the date of filing of the application
although capable of being worked without or three (3) years from the date of the patent,
• Requirement to Obtain a License satisfactory reason whichever period expires last

Section 95. Requirement to Obtain a License on Reasonable Commercial Terms. - See Sections 93.2, 93.3, 93.4 and 97 of the Anytime after the grant of the patent
95.1. The license will only be granted after the petitioner has made efforts to obtain IPC
authorization from the patent owner on reasonable commercial terms and conditions but
such efforts have not been successful within a reasonable period of time.

95.2. The requirement under Subsection 95.1 shall not apply in the following cases:
(a) Where the petition for compulsory license seeks to remedy a practice determined after
judicial or administrative process to be anti-competitive;
(b) In situations of national emergency or other circumstances of extreme urgency; NOTES:
(c) In cases of public non-commercial use. • FRANCHISE – type of license that a party (franchisee) acquires to allow them to have access
to a business’ (franchisor) proprietary knowledge, processes and trademarks in order to allow
the party to sell a product/provide a service under the business’ name
95.3. In situations of national emergency or other circumstances of extreme urgency, the right
holder shall be notified as soon as reasonably practicable. • FRANCHISE AGREEMENT – legal, binding contract between a franchisor and franchisee
• PATENT TROLL – company that purchases a patent, often form a bankrupt firm, and then
95.4. In the case of public non-commercial use, where the government or contractor, without sues another company by claiming that one of its products infringes on the purchased patent
making a patent search, knows or has demonstrable grounds to know that a valid patent is or • ORPHAN WORK – a copyright protected work for which right holders are positively
will be used by or for the government, the right holder shall be informed promptly. (n) indeterminate or uncontactable. Sometimes the names of the originators/right holders are
known, yet it is impossible to contact them because additional details cannot be found.

VOLUNTARY LICENSING COMPULSORY LICENSING


As to Grantor
Patent Owner Director of Legal Affairs

As to Grounds
No specific grounds Those mentioned under Section 10 of RA

! DomingoŸAriolaŸMartinezŸMolaer

DAMM NOTES – LAW ON INTELLECTUAL PROPERTY SY $%&! - !"#!
Atty. Castillo-Taleon


An invention that can be produced and used in any industry shall be industrially applicable
PATENTS (IPC, Sec. 27).
(5) PATENTABLE SUBJECT MATTER
An invention that does not fall within the prohibitions of a non-patentable invention under
PATENT is the right granted to an inventor by the State, or by the regional office acting for Sec. 22 of the IPC.
several States, which allows the inventor to exclude anyone else from commercially exploiting
his invention for a limited period (WIPO).
• It is the statutory monopoly which protects against unlicensed use of the patented device or
process even by the one who discovered it through independent research. Section 21. Patentable Inventions. - Any technical solution of a problem in any field of human
• TERM OF PATENT: 20 years from filing date of application. Upon expiration, invention inures activity which is new, involves an inventive step and is industrially applicable shall be patentable.
to the people. It may be, or may relate to, a product, or process, or an improvement of any of the foregoing.

PATENT COPYRIGHT
As to time of accrual Patentable inventions may be/may relate to:
When a person, by independent research It may be vested in a work closely similar or • A product – a machine, device, an article of manufacture, a composition of matter, a
arrives at the same product or that is already even identical to an earlier, or already microorganism
patented, he is restrained by the arm of the patented work, provided that the former is truly • A process – method of use, method of manufacturing, non-biological process, microbiological
law for exploiting such an invention by reason original, i.e. it owes its existence to its creator. process
of the patent already granted to the earlier • Computer related inventions
discoverer. • An improvement of any of the foregoing

As to non-patentable inventions
May NOT be subject of a patent MAY be subject of copyright
Section 23. Novelty. - An invention shall not be considered new if it forms part of a prior art.

No Patent, No Protection Section 24. Prior Art. - Prior art shall consist of:
The ultimate goal of a patent system is to bring new designs and technologies into the public 24.1. Everything which has been made available to the public anywhere in the world, before the
domain through disclosure. Ideas, once disclosed to the public without the protection of a valid filing date or the priority date of the application claiming the invention; and
patent, are subject to appropriation without significant restraint (Pearl & Dean (Phil.) vs
Shoemart, GR No. 148222). 24.2. The whole contents of an application for a patent, utility model, or industrial design
registration, published in accordance with this Act, filed or effective in the Philippines, with a
filing or priority date that is earlier than the filing or priority date of the application: Provided, That
I. PATENTABLE INVENTIONS the application which has validly claimed the filing date of an earlier application under Section 31
of this Act, shall be prior art with effect as of the filing date of such earlier application: Provided
PATENTABLE INVENTION refers to any technical solution of a problem in any field of human further, That the applicant or the inventor identified in both applications are not one and the
activity which is new, involves an inventive step and is industrially applicable (IPC, Sec. 21). same.
Requisites of Patentability (TINI-P)
(1) Any TECHNICAL SOLUTION of a problem in any field of human activity; Section 25. Non-Prejudicial Disclosure. -
(2) INVENTIVE STEP 25.1. The disclosure of information contained in the application during the twelve (12) months
An invention involves an inventive step if, having regard to prior art, it is not obvious to a preceding the filing date or the priority date of the application shall not prejudice the applicant on
person skilled in the art at the time of the filing date or priority date of the application claiming the ground of lack of novelty if such disclosure was made by:
the invention (IPC, Sec. 26). (a) The inventor;
(3) NOVELTY (b) A patent office and the information was contained (a) in another application filed by the
An invention shall not be considered new if it forms part of a prior art (IPC, Sec. 23). inventor and should not have been disclosed by the office, or (b) in an application filed
(4) INDUSTRIAL APPLICABILITY


! DomingoŸAriolaŸMartinezŸMolaer

DAMM NOTES – LAW ON INTELLECTUAL PROPERTY SY $%&! - !"#!
Atty. Castillo-Taleon


without the knowledge or consent of the inventor by a third party which obtained the (1) Discoveries. scientific theories and mathematical methods, a law of nature, a scientific truth
information directly or indirectly from the inventor; or or knowledge as such
(c) A third party which obtained the information directly or indirectly from the inventor. (2) Abstract ides/theories, fundamental concepts apart from the means or processes for
carrying the concept to produce a technical effect
25.2. For the purposes of Subsection 25.1, "inventor" also means any person who, at the filing (3) Schemes, rules, and methods of performing mental acts and playing games
date of application, had the right to the patent. (4) Method of doing business, such as a method or system for transacting business without the
technical means for carrying out the method or system
(5) Methods for treatment of the human or animal boyd by surgery or therapy and diagnostic
Section 26. Inventive Step. - An invention involves an inventive step if, having regard to prior methods practiced on the human or animal body. The non-patentability shall not apply to
art, it is not obvious to a person skilled in the art at the time of the filing date or priority date of products and compositions for use in any of these methods
the application claiming the invention. (6) Anything which is contrary to public order, healthy, welfare, or morality, or process for
cloning or modifying the germ line genetic identity of humans or animals or uses of the
human embryo
Section 27. Industrial Applicability. - An invention that can be produced and used in any (7) Aesthetic creations
industry shall be industrially applicable. (8) Programs for computers
(9) Plant varieties or animal breeds or essentially biological processes for the production of
plans and animals. This provision shall not apply to microorganisms and non-biological and
microbiological processes.

II. NON-PATENTABLE INVENTIONS


III. OWNERSHIP OF A PATENT
Section 22. Non-Patentable Inventions. - The following shall be excluded from patent
A. Right to a Patent
protection:
22.1. Discoveries, scientific theories and mathematical methods;
Section 28. Right to a Patent. - The right to a patent belongs to the inventor, his heirs, or
22.2. Schemes, rules and methods of performing mental acts, playing games or doing business, assigns. When two (2) or more persons have jointly made an invention, the right to a patent shall
and programs for computers; belong to them jointly.

22.3 Methods for treatment of the human or animal body by surgery or therapy and diagnostic
methods practiced on the human or animal body. This provision shall not apply to products and
composition for use in any of these methods;
B. First-to-File Rule
22.4. Plant varieties or animal breeds or essentially biological process for the production of
plants or animals. This provision shall not apply to micro-organisms and non-biological and Section 29. First to File Rule. - If two (2) or more persons have made the invention separately
microbiological processes. and independently of each other, the right to the patent shall belong to the person who filed an
application for such invention, or where two or more applications are filed for the same invention,
Provisions under this subsection shall not preclude Congress to consider the enactment of a law to the applicant who has the earliest filing date or, the earliest priority date.
providing sui generis protection of plant varieties and animal breeds and a system of community
intellectual rights protection:

22.5. Aesthetic creations; and


FIRST-TO-FILE RULE grants the patent rights of an invention to the person who first files a
patent application for that invention, whether or not he is the inventor.
22.6. Anything which is contrary to public order or morality.
FIRST-TO-INVENT RULE grants the patent rights to the first true and actual inventor, his heirs,
legal representatives or assigns.
2 2
Non-Patentable Inventions (DAS-M AAP ) NOTE: The Philippines adheres to the first-to-file rule.


! DomingoŸAriolaŸMartinezŸMolaer

DAMM NOTES – LAW ON INTELLECTUAL PROPERTY SY $%&! - !"#!
Atty. Castillo-Taleon


duties, unless agreed otherwise

C. Inventions Created Pursuant to a Commission

Section 30. Inventions Created Pursuant to a Commission. -


30.1. The person who commissions the work shall own the patent, unless otherwise provided in
the contract. IV. GROUNDS FOR CANCELLATION OF A PATENT

30.2. In case the employee made the invention in the course of his employment contract, the Section 61. Cancellation of Patents. -
patent shall belong to: 61.1. Any interested person may, upon payment of the required fee, petition to cancel the patent
(a) The employee, if the inventive activity is not a part of his regular duties even if the employee or any claim thereof, or parts of the claim, on any of the following grounds:
uses the time, facilities and materials of the employer. (a) That what is claimed as the invention is not new or patentable;
(b) The employer, if the invention is the result of the performance of his regularly-assigned (b) That the patent does not disclose the invention in a manner sufficiently clear and complete
duties, unless there is an agreement, express or implied, to the contrary. for it to be carried out by any person skilled in the art; or
(c) That the patent is contrary to public order or morality.

61.2. Where the grounds for cancellation relate to some of the claims or parts of the claim,
cancellation may be effected to such extent only.
D. Right of Priority

Section 31. Right of Priority. - An application for patent filed by any person who has previously Effect: Termination of rights conferred by the patent (IPC, Sec. 66).
applied for the same invention in another country which by treaty, convention, or law affords
similar privileges to Filipino citizens, shall be considered as filed as of the date of filing the
foreign application: Provided, That:
(a) the local application expressly claims priority; V. REMEDY OF THE TRUE AND ACTUAL INVENTOR
(b) it is filed within twelve (12) months from the date the earliest foreign application was filed;
and Section 29 (see First-to-File Rule)
(c) a certified copy of the foreign application together with an English translation is filed within
six (6) months from the date of filing in the Philippines. Section 67. Patent Application by Persons Not Having the Right to a Patent. –
67.1. If a person referred to in Section 29 other than the applicant, is declared by final court
order or decision as having the right to the patent, such person may, within three (3) months
Summary of Rules after the decision has become final:
STATUS PERSONS ENTITLED TO PATENT (a) Prosecute the application as his own application in place of the applicant;
Two (2) or more persons invent separately He who first files (b) File a new patent application in respect of the same invention;
and independently (c) Request that the application be refused; or
(d) Seek cancellation of the patent, if one has already been issued.
Two (2) or more applications filed for the same Applicant who has the earliest filing date or
invention earliest priority date 67.2. The provisions of Subsection 38.2 shall apply mutatis mutandis to a new application filed
under Subsection 67.1(b).
Inventions created pursuant to a commission Person who commissioned the work, unless
agreed otherwise
Section 68. Remedies of the True and Actual Inventor. - If a person, who was deprived of the
patent without his consent or through fraud is declared by final court order or decision to be the
Employee makes the invention in the course EMPLOYEE, if invention is not part of his
true and actual inventor, the court shall order for his substitution as patentee, or at the option of
of employment regular duties even if he uses the time,
facilities and materials of the employer the true inventor, cancel the patent, and award actual and other damages in his favor if
warranted by the circumstances.
EMPLOYER, if the invention is the result of
the performance of his regularly assigned

! DomingoŸAriolaŸMartinezŸMolaer

DAMM NOTES – LAW ON INTELLECTUAL PROPERTY SY $%&! - !"#!
Atty. Castillo-Taleon


Section 70. Time to File Action in Court. - The actions indicated in Sections 67 and 68 shall VII. LIMITATIONS OF PATENT RIGHTS
be filed within one (1) year from the date of publication made in accordance with Sections 44
and 51, respectively. Section 71. Rights Conferred by Patent. -
71.1. A patent shall confer on its owner the following exclusive rights:
(a) Where the subject matter of a patent is a product, to restrain, prohibit and prevent any
unauthorized person or entity from making, using, offering for sale, selling or importing that
product;
(b) Where the subject matter of a patent is a process, to restrain, prevent or prohibit any
unauthorized person or entity from using the process, and from manufacturing, dealing in,
VI. RIGHTS CONFERRED BY A PATENT using, selling or offering for sale, or importing any product obtained directly or indirectly from
such process.
Section 71. Rights Conferred by Patent. -
71.2. Patent owners shall also have the right to assign, or transfer by succession the patent, and
71.1. A patent shall confer on its owner the following exclusive rights:
to conclude licensing contracts for the same.
(a) Where the subject matter of a patent is a product, to restrain, prohibit and prevent any
unauthorized person or entity from making, using, offering for sale, selling or importing that
product;
Section 72. Limitations of Patent Rights. - The owner of a patent has no right to prevent third
(b) Where the subject matter of a patent is a process, to restrain, prevent or prohibit any
parties from performing, without his authorization, the acts referred to in Section 71 hereof in the
unauthorized person or entity from using the process, and from manufacturing, dealing in,
following circumstances:
using, selling or offering for sale, or importing any product obtained directly or indirectly from
72.1 Using a patented product which has been put on the market in the Philippines by the owner
such process.
of the product, or with his express consent, insofar as such use is performed after that product
has been so put on the said market;
71.2. Patent owners shall also have the right to assign, or transfer by succession the patent, and
to conclude licensing contracts for the same.
72.2. Where the act is done privately and on a non-commercial scale or for a non-commercial
purpose: Provided, That it does not significantly prejudice the economic interests of the owner of
the patent;
Section 55. Annual Fees. -
55.1. To maintain the patent application or patent, an annual fee shall be paid upon the
72.3. Where the act consists of making or using exclusively for the purpose of experiments that
expiration of four (4) years from the date the application was published pursuant to Section 44
relate to the subject matter of the patented invention;
hereof, and on each subsequent anniversary of such date. Payment may be made within three
(3) months before the due date. The obligation to pay the annual fees shall terminate should the
72.4. Where the act consists of the preparation for individual cases, in a pharmacy or by a
application be withdrawn, refused, or cancelled.
medical professional, of a medicine in accordance with a medical prescription or acts concerning
the medicine so prepared;
55.2. If the annual fee is not paid, the patent application shall be deemed withdrawn or the
patent considered as lapsed from the day following the expiration of the period within which the
72.5. Where the invention is used in any ship, vessel, aircraft, or land vehicle of any other
annual fees were due. A notice that the application is deemed withdrawn or the lapse of a patent
country entering the territory of the Philippines temporarily or accidentally: Provided, That such
for non-payment of any annual fee shall be published in the IPO Gazette and the lapse shall be
invention is used exclusively for the needs of the ship, vessel, aircraft, or land vehicle and not
recorded in the Register of the Office.
used for the manufacturing of anything to be sold within the Philippines.
55.3. A grace period of six (6) months shall be granted for the payment of the annual fee, upon
payment of the prescribed surcharge for delayed payment.

A. Prior Use

Section 73. Prior User. -


73.1. Notwithstanding Section 72 hereof, any prior user, who, in good faith was using the
invention or has undertaken serious preparations to use the invention in his enterprise or
business, before the filing date or priority date of the application on which a patent is granted,


! DomingoŸAriolaŸMartinezŸMolaer

DAMM NOTES – LAW ON INTELLECTUAL PROPERTY SY $%&! - !"#!
Atty. Castillo-Taleon


shall have the right to continue the use thereof as envisaged in such preparations within the
territory where the patent produces its effect. 76.6. Anyone who actively induces the infringement of a patent or provides the infringer with a
component of a patented product or of a product produced because of a patented process
73.2. The right of the prior user may only be transferred or assigned together with his enterprise knowing it to be especially adopted for infringing the patented invention and not suitable for
or business, or with that part of his enterprise or business in which the use or preparations for substantial non-infringing use shall be liable as a contributory infringer and shall be jointly and
use have been made. severally liable with the infringer.

A. Jurisdiction
B. Use by the Government
AM No. 02-1-11-SC dated February 19, 2002
Section 74. Use of Invention by Government. -
74.1. A Government agency or third person authorized by the Government may exploit the RE: DESIGNATION OF AN INTELLECTUAL PROPERTY JUDGE FOR MANILA.
invention even without agreement of the patent owner where:
(a) the public interest, in particular, national security, nutrition, health or the development of (A) Acting on the memorandum dated 15 January 2002 of Deputy Court Administrator
other sectors, as determined by the appropriate agency of the government, so requires; or Christopher O. Lock, favorably endorsed by Court Administrator Presbitero J. Velasco, Jr.,
(b) A judicial or administrative body has determined that the manner of exploitation, by the the Court hereby designates Branch 24 of the Regional Trial Court of Manila, presided over
owner of the patent or his licensee, is anti-competitive. by Judge ANTONIO M. EUGENIO, JR., as Special Intellectual Property Court for Manila in
substitution of Branch 1 of said Court which was designated Special Intellectual Property
74.2. The use by the Government, or third person authorized by the Government shall be Court, then presided over by Judge Rebecca de Guia Salvador (now Associate Justice of
subject, mutatis mutandis, to the conditions set forth in Sections 95 to 97 and 100 to 102. the Court of Appeals), pursuant to Administrative Order No. 113-95 dated 5 October 1995.

As judge of the Special Intellectual Property Court, Judge Antonio M. Eugenio, Jr. shall try
and decide cases involving violations of intellectual property rights under the Intellectual
Property Code (R.A. No. 8293) committed within the City of Manila.
VIII. PATENT INFRINGEMENT
The earlier designation of Branch 1 of the Regional Trial Court of Manila as a Special
Intellectual Property Court is hereby REVOKED, and the cases involving violations of
Section 76. Civil Action for Infringement. -
intellectual property rights earlier assigned to said Branch 1 pursuant to and by virtue of
76.1. The making, using, offering for sale, selling, or importing a patented product or a product
Administrative Order No. 113-95 are hereby re-assigned to Branch 24 of said Court.
obtained directly or indirectly from a patented process, or the use of a patented process without
the authorization of the patentee constitutes patent infringement.
(B) Furthermore, acting on the recommendation of Hon. Associate Justice Reynato S. Puno,
76.2. Any patentee, or anyone possessing any right, title or interest in and to the patented
Chairman of the Committee on Revision of the Rules of Court, and the Office of the Court
invention, whose rights have been infringed, may bring a civil action before a court of competent
Administrator, and in order to ensure speedy disposition of cases involving violations of
jurisdiction, to recover from the infringer such damages sustained thereby, plus attorney’s fees
intellectual property rights under the Intellectual Property Code (R.A. No. 8293), the
and other expenses of litigation, and to secure an injunction for the protection of his rights.
following Regional Trial Courts (RTCs) are hereby designated Special Intellectual Property
Courts:
76.3. If the damages are inadequate or cannot be readily ascertained with reasonable certainty,
the court may award by way of damages a sum equivalent to reasonable royalty.
Region 1: RTC, Legazpi City
(1) Hon. Antonio M. Esteves (2) Hon. Filemon B. Montenegro
76.4. The court may, according to the circumstances of the case, award damages in a sum
Presiding Judge, Branch 5 Presiding Judge, Branch 26
above the amount found as actual damages sustained: Provided, That the award does not
RTC, Baguio City RTC, Naga City
exceed three (3) times the amount of such actual damages.
(2) Hon. Manuel L. Argel
Presiding Judge, Branch Region 8:
76.5. The court may, in its discretion, order that the infringing goods, materials and implements
65 Hon. Frisco T. Lilagan
predominantly used in the infringement be disposed of outside the channels of commerce or
RTC, Laoag City Presiding Judge, Branch 34
destroyed, without compensation.
RTC, Tacloban City

! DomingoŸAriolaŸMartinezŸMolaer

DAMM NOTES – LAW ON INTELLECTUAL PROPERTY SY $%&! - !"#!
Atty. Castillo-Taleon


Region 2: damages, be criminally liable therefor and, upon conviction, shall suffer imprisonment for the
Hon. Rolando R. Velasco Region 12: period of not less than six (6) months but not more than three (3) years and/or a fine of not less
Presiding Judge, Branch 6 Hon. Albert B. Abragan than One hundred thousand pesos (P100,000) but not more than Three hundred thousand
RTC, Aparri, Cagayan Presiding Judge, Branch 3 pesos (P300,000), at the discretion of the court. The criminal action herein provided shall
RTC, Iligan City prescribed in three (3) years from date of the commission of the crime.
Region 5:
(1) Hon. Vladimir B. Bruselas
Presiding Judge, Branch 6

The foregoing Special Intellectual Property Courts shall try and decide cases involving C. Tests in Patent Infringement
violations of intellectual property rights defined under the Intellectual Property Code
committed within their respective territorial areas.Since there are only a few cases of (1) Literal Infringement
violations of intellectual property rights now pending in other branches of the
aforementioned Regional Trial Courts, such cases shall remain with and shall be decided
Section 75.1. The extent of protection conferred by the patent shall be determined by the
by the branches to which they have been assigned. Only cases hereafter filed may be
claims, which are to be interpreted in the light of the description and drawings.
assigned to the above designated special courts.

(C) Finally, in order to ensure a just and equitable distribution of cases among the Judges (2) Doctrine of Equivalents
concerned, all the aforementioned Special Intellectual Property Courts shall continue to Section 75.2. For the purpose of determining the extent of protection conferred by the
participate in the raffles of other cases: Provided, however, that the Executive Judges patent, due account shall be taken of elements which are equivalent to the elements expressed
concerned shall adopt a procedure whereby every intellectual property right case assigned in the claims, so that a claim shall be considered to cover not only all the elements as expressed
to a Special Intellectual Property Court should be considered a case raffled to it and be duly therein, but also equivalents.
credited to such court.

Samson vs Cabanos (GR No. 161693, June 28, 2005)


The petitioner, owner/proprietor of ITTI Shoes/Mano Shoes Manufactuirng Corporation, allegedly D. Defenses in Action for Infringement
sold or offers the sale of garment product using the trademark “Caterpillar” to the prejudice of
Caterpillar, Inc., private respondent in this case. The respondent filed the case with the RTC.
The petitioner questioned the jurisdiction of the trial court over the offense charged contending Section 81. Defenses in Action for Infringement. - In an action for infringement, the
that the case should be filed with the MTC because violation of unfair competition is penalized defendant, in addition to other defenses available to him, may show the invalidity of the patent,
with imprisonment not exceeding 6 years under RA 7691. or any claim thereof, on any of the grounds on which a petition of cancellation can be brought
under Section 61 hereof.
The SC held that under Section 163 of the IPC, actions for unfair competition shall be
brought before the proper courts with appropriate jurisdiction under existing laws. The law
contemplated in Section 163 of IPC is RA 166 otherwise known as the Trademark Law. Section
27 of the Trademark Law provides that jurisdiction over cases for infringement of registered
marks, unfair competition, false designation of origin and false description or representation, is E. Contributory Patent Infringement
lodged with the Court of First Instance (now Regional Trial Court). Since RA 7691 is a general
law and IPC in relation to Trademark Law is a special law, the latter shall prevail. Actions for Section 75.1. The extent of protection conferred by the patent shall be determined by the
unfair competition therefore should be filed with the RTC. claims, which are to be interpreted in the light of the description and drawings.

B. Criminal Action for Patent Infringement

Section 84. Criminal Action for Repetition of Infringement. - If infringement is repeated by


the infringer or by anyone in connivance with him after finality of the judgment of the court IX. ASSIGNMENT AND TRANSMISSION OF RIGHTS
against the infringer, the offenders shall, without prejudice to the institution of a civil action for


!" DomingoŸAriolaŸMartinezŸMolaer

DAMM NOTES – LAW ON INTELLECTUAL PROPERTY SY $%&! - !"#!
Atty. Castillo-Taleon


Section 103. Transmission of Rights. -
103.1 Patents or applications for patents and invention to which they relate, shall be protected in
the same way as the rights of other property under the Civil Code. I. DEFINITION OF MARKS, COLLECTIVE MARKS, TRADE NAMES, SLOGAN

103.2. Inventions and any right, title or interest in and to patents and inventions covered thereby,
may be assigned or transmitted by inheritance or bequest or may be the subject of a license Section 121. Definitions. - As used in Part III, the following terms have the following meanings:
contract. 121.1. "Mark" means any visible sign capable of distinguishing the goods (trademark) or
services (service mark) of an enterprise and shall include a stamped or marked container of
goods;
Section 104. Assignment of Inventions. - An assignment may be of the entire right, title or
interest in and to the patent and the invention covered thereby, or of an undivided share of the 121.2. "Collective mark" means any visible sign designated as such in the application for
entire patent and invention, in which event the parties become joint owners thereof. An registration and capable of distinguishing the origin or any other common characteristic,
assignment may be limited to a specified territory. including the quality of goods or services of different enterprises which use the sign under the
control of the registered owner of the collective mark;

Section 105. Form of Assignment. - The assignment must be in writing, acknowledged before 121.3. "Trade name" means the name or designation identifying or distinguishing an enterprise;
a notary public or other officer authorized to administer oath or perform notarial acts, and
certified under the hand and official seal of the notary or such other officer. 121.4. "Bureau" means the Bureau of Trademarks;

121.5. "Director" means the Director of Trademarks;

121.6. "Regulations" means the Rules of Practice in Trademarks and Service Marks formulated
Section 106. Recording. - by the Director of Trademarks and approved by the Director General; and
106.1. The Office shall record assignments, licenses and other instruments relating to the
transmission of any right, title or interest in and to inventions, and patents or application for 121.7. "Examiner" means the trademark examiner.
patents or inventions to which they relate, which are presented in due form to the Office for
registration, in books and records kept for the purpose. The original documents together with a
signed duplicate thereof shall be filed, and the contents thereof should be kept confidential. If the
original is not available, an authenticated copy thereof in duplicate may be filed. Upon recording, MARK is any visible sign capable of distinguishing the goods (trademark) or services (service
the Office shall retain the duplicate, return the original or the authenticated copy to the party who mark) of an enterprise and shall include a stamped or marked container of goods (IPC, Sec.
filed the same and notice of the recording shall be published in the IPO Gazette. 121.1).

106.2. Such instruments shall be void as against any subsequent purchaser or mortgagee for
valuable consideration and without notice, unless, it is so recorded in the Office, within three (3) Kinds of Mark:
months from the date of said instrument, or prior to the subsequent purchase or mortgage. (1) TRADEMARK is any distinctive words, letters, numerals, drawings, pictures, shapes,
colors, logotypes, labels or combinations used to distinguish goods or services.
(2) SERVICE MARK is a mark used to distinguish certain services as those provided by a
Section 107. Rights of Joint Owners. - If two (2) or more persons jointly own a patent and the specific enterprise.
invention covered thereby, either by the issuance of the patent in their joint favor or by reason of (3) WELL-KNOWN MARK refers to a mark considered well-known by the competent authority
the assignment of an undivided share in the patent and invention or by reason of the succession and of the country where the protection for the mark is sought.
in title to such share, each of the joint owners shall be entitled to personally make, use, sell, or (4) COLLECTIVE MARK refers to any visible sign, designated as such in the application for
import the invention for his own profit: Provided, however, That neither of the joint owners shall registration and capable of distinguishing the origin or any other common characteristics,
be entitled to grant licenses or to assign his right, title or interest or part thereof without the including the quality of goods or services of different enterprises which use the sign under
consent of the other owner or owners, or without proportionally dividing the proceeds with such the control of the registered owner of the collective mark.
other owner or owners. (5) SLOGAN refers to short words or phrases that capture a company’s brand essence,
personality, and positioning, and distinguish the firm from competitors.

TRADEMARKS Under the IPC, slogans are protected as word marks.



!! DomingoŸAriolaŸMartinezŸMolaer

DAMM NOTES – LAW ON INTELLECTUAL PROPERTY SY $%&! - !"#!
Atty. Castillo-Taleon


II. ACQUISITION OF OWNERSHIP OF MARK

Functions of a Trademark
(1) To indicate the origin of the goods to which they are attached; Section 122. How Marks are Acquired. - The rights in a mark shall be acquired through
(2) To guarantee the standard of quality of the goods; and registration made validly in accordance with the provisions of this law.
(3) To advertise the goods.

TRADEMARK TRADENAME
BIRKENSTOCK ORTHOPAEDIE GMBH AND CO. KG (formerly BIRKENSTOCK
As to separate existence
ORTHOPAEDIE GMBH) vs PHILIPPINE SHOE EXPO MARKETING CORPORATION
Has an existence distinct from the existence of Attached to the natural or juridical person who G.R. No. 194307
the proprietor or judicial person doing does business and produces the goods or
business and producing the goods or the services. The cancellation of the trademark registration due to failure to file the Declaration of Use is
services offered by such person or enterprise. tantamount to a waiver of the registrant’s right or interest over the trademark and thus, will not
preclude another party’s subsequent application of the mark. A trademark registration merely
As to examples creates a prima facie presumption of ownership which yields to superior evidence of actual and
The Nike “swoosh” or the check mark looking Nike or Coca-Cola (the latter is also an real ownership of a trademark.
logo or “Big Mac” example of the trade name used as a
trademark) FACTS:
As to purpose Birkenstock Orthopaedie GMBH (Birkenstock Orthopaedie) applied for the trademark registration
Designates the goods or services offered by Identifies and distinguishes an enterprise. of “BIRKENSTOCK” with the IPO. Philippine Shoe Expo opposed the application on account of
person or enterprise. prior use for more than 16 years and registration of the mark “BIRKENSTOCK AND DEVICE”.
The Bureau of Legal Affairs (BLA) sustained the opposition because: (i) the competing marks
As to registration are confusingly similar and are used on the same and related goods; (ii) Birkenstock
Must be registered in order to secure No need to register in order to secure Orthopaedie failed to prove actual use of the mark in trade and business in the Philippines; (iii)
protection for them. Trademarks are registered protection for them. Trade names are prior right over the mark was not lost even though the registration of “BIRKENSTOCK AND
before the IPO. registered before the DTI. DEVICE” was cancelled, as there was proof of the mark’s continuous and uninterrupted use in
commerce in the Philippines; and (iv) “BIRKENSTOCK” is not well -known in the Philippines and
As to transferability internationally. The IPO Director General set aside the BLA’s ruling. The IPO Director General
May be transferred with or without transfer of Change of ownership of trade name must be found Birkenstock Orthopaedie to be the true and lawful owner and prior user of the
the business. made with transfer of enterprise or part “BIRKENSTOCK” marks. The IPO Director General further held that “BIRKENSTOCK AND
thereof. DEVICE” is no longer an impediment to the registration of “BIRKENSTOCK” as the former’s
registration had been cancelled on account of the registrant’s failure to file the 10th year
Declaration of Use. The Court of Appeals reinstated the decision of the BLA.
In practice, a word, a name or a phrase, coupled with indicators of business organization, such
as “Inc.”, “Corp.” or “Co.” will not be registered as trademarks or service marks. RULING:
The Supreme Court ruled in favor of Birkenstock Orthopaedie for the following reasons:
The IPC no longer provides for prior use as a condition for ownership of a mark (IPC, Sec. 122). (i) Under the former trademark law, Republic Act 166, failure to file the Declaration of Use
Registration is the sole basis. However, a trade name or business name may be acquired by results in the automatic cancellation of the trademark which in turn is tantamount to the
prior use and need not be registered. abandonment or withdrawal of the registrant’s right or interest over the trade mark.
Applying this rule, the registrant is deemed to have abandoned its right or interest over the
mark “BIRKENSTOCK AND DEVICE” on account of its failure to file the 10th year
Declaration of Use; and
(ii) Birkenstock Orthopaedie proved its true and lawful ownership of the mark “Birkenstock”.

Evidence was submitted on (i) the use of the mark in Europe since 1774, when its inventor,
Johann Birkenstock, used the mark on his line of quality footwear, which use was continued by
numerous generations of his kin; and (ii) the worldwide registration of the mark
“BIRKENSTOCK”. The Supreme Court did not find credible the evidence of Philippine Shoe

!" DomingoŸAriolaŸMartinezŸMolaer

DAMM NOTES – LAW ON INTELLECTUAL PROPERTY SY $%&! - !"#!
Atty. Castillo-Taleon


Expo as it was able to submit only copies of sales invoices and advertisements, which showed TRADE NAME/ BUSINESS IDENTIFIER means the name or designation identifying or
merely its transactions involving the same. The Supreme Court found the registration of distinguishing an enterprise; any individual name or surname, firm name, device or word used by
“BIRKENSTOCK AND DEVICE” to have been done in bad faith and found it highly incredible manufacturers, industrialists, merchants, and others to identify their business, vocations or
that Philippine Shoe Expo came up on its own with the mark “BIRKENSTOCK”, obviously of occupations.
German origin and a highly distinct and arbitrary mark. The Supreme Court pointed out that
Philippine Shoe Expo obviously knew of the existence of “BIRKENSTOCK” and its use by A trade name need not be registered with the IPO before an infringement suit may be filed by its
Birkenstock Orthopaedie and that it clearly intended to take advantage of the goodwill generated owner against the owner of an infringing trademark. All that is required is that the trade name is
by the “BIRKENSTOCK” mark. previously used in trade or commerce in the Philippines.

Finally, the Supreme Court reiterated the principle that registration of the trademark merely
creates a prima facie presumption of ownership which yields to superior evidence of actual and
real ownership of a trademark. In the words of the Supreme Court:
“Clearly, it is not the application or registration of a trademark that vests ownership thereof, but it
is the ownership of a trademark that confers the right to register the same. A trademark is an IV. NON-REGISTRABLE MARKS
industrial property over which its owner is entitled to property rights which cannot be
appropriated by unscrupulous entities that, in one way or another, happen to register such
trademark ahead of its true and lawful owner. The presumption of ownership accorded to a Section 123. Registrability. -
registrant must then necessarily yield to superior evidence of actual and real ownership of a 123.1. A mark cannot be registered if it:
trademark.” (a) Consists of immoral, deceptive or scandalous matter, or matter which may disparage or
falsely suggest a connection with persons, living or dead, institutions, beliefs, or national
symbols, or bring them into contempt or disrepute;

(b) Consists of the flag or coat of arms or other insignia of the Philippines or any of its political
III. ACQUISITION OF OWNERSHIP OF TRADE NAME subdivisions, or of any foreign nation, or any simulation thereof;

(c) Consists of a name, portrait or signature identifying a particular living individual except by
Section 165. Trade Names or Business Names. - his written consent, or the name, signature, or portrait of a deceased President of the
165.1. A name or designation may not be used as a trade name if by its nature or the use to Philippines, during the life of his widow, if any, except by written consent of the widow;
which such name or designation may be put, it is contrary to public order or morals and if, in
particular, it is liable to deceive trade circles or the public as to the nature of the enterprise (d) Is identical with a registered mark belonging to a different proprietor or a mark with an
identified by that name. earlier filing or priority date, in respect of:
(i) The same goods or services, or
165.2. (a) Notwithstanding any laws or regulations providing for any obligation to register trade (ii) Closely related goods or services, or
names, such names shall be protected, even prior to or without registration, against any unlawful (iii) If it nearly resembles such a mark as to be likely to deceive or cause confusion;
act committed by third parties.
(e) Is identical with, or confusingly similar to, or constitutes a translation of a mark which is
(b) In particular, any subsequent use of the trade name by a third party, whether as a trade considered by the competent authority of the Philippines to be well-known internationally
name or a mark or collective mark, or any such use of a similar trade name or mark, likely to and in the Philippines, whether or not it is registered here, as being already the mark of a
mislead the public, shall be deemed unlawful. person other than the applicant for registration, and used for identical or similar goods or
services: Provided, That in determining whether a mark is well-known, account shall be
165.3. The remedies provided for in Sections 153 to 156 and Sections 166 and 167 shall apply taken of the knowledge of the relevant sector of the public, rather than of the public at
mutatis mutandis. large, including knowledge in the Philippines which has been obtained as a result of the
promotion of the mark;
165.4. Any change in the ownership of a trade name shall be made with the transfer of the
enterprise or part thereof identified by that name. The provisions of Subsections 149.2 to 149.4 (f) Is identical with, or confusingly similar to, or constitutes a translation of a mark considered
shall apply mutatis mutandis. well-known in accordance with the preceding paragraph, which is registered in the
Philippines with respect to goods or services which are not similar to those with respect to
which registration is applied for: Provided, That use of the mark in relation to those goods
or services would indicate a connection between those goods or services, and the owner of


!" DomingoŸAriolaŸMartinezŸMolaer

DAMM NOTES – LAW ON INTELLECTUAL PROPERTY SY $%&! - !"#!
Atty. Castillo-Taleon


the registered mark: Provided further, That the interests of the owner of the registered mark (4) Is IDENTICAL with a registered mark belonging to a different proprietor or a mark with an
are likely to be damaged by such use; earlier filing or priority date, in respect of:
a. The same goods or services, or
(g) Is likely to mislead the public, particularly as to the nature, quality, characteristics or b. Closely related goods or services, or
geographical origin of the goods or services; c. If it nearly resembles such a mark as to be likely to deceive or cause confusion;
(5) Is IDENTICAL with, or confusingly similar to, or constitutes a translation of a mark which
(h) Consists exclusively of signs that are generic for the goods or services that they seek to is considered by the competent authority of the Philippines to be well-known
identify; internationally and in the Philippines, whether or not it is registered here, as being already
the mark of a person other than the applicant for registration, and used for identical or
(i) Consists exclusively of signs or of indications that have become customary or usual to similar goods or services;
designate the goods or services in everyday language or in bona fide and established trade (6) IDENTICAL with, or confusingly similar to, or constitutes a translation of a mark
practice; considered well-known in accordance with the preceding paragraph, which is registered in
the Philippines with respect to goods or services which are not similar to those with
(j) Consists exclusively of signs or of indications that may serve in trade to designate the kind, respect to which registration is applied for, provided that:
quality, quantity, intended purpose, value, geographical origin, time or production of the a. Use of the mark in relation to those goods or services would indicate a connection
goods or rendering of the services, or other characteristics of the goods or services; between those goods or services, and the owner of the registered mark; and
b. That the interests of the owner of the registered mark are likely to be damaged by
(k) Consists of shapes that may be necessitated by technical factors or by the nature of the such use.
goods themselves or factors that affect their intrinsic value; (7) Is likely to MISLEAD the public, particularly as to the nature, quality, characteristics or
geographical origin of the goods or services;
(l) Consists of color alone, unless defined by a given form; or (8) Consists exclusively of signs that are GENERIC for the goods or services that they seek
to identify;
(m) Is contrary to public order or morality. (9) Consists exclusively of signs or of indications that have become CUSTOMARY or usual to
designate the goods or services in everyday language or in bona fide and established
trade practices;
123.2. As regards signs or devices mentioned in paragraphs (j), (k), and (l), nothing shall prevent (10) Consists exclusively of signs or of indications that may serve in trade to DESIGNATE the
the registration of any such sign or device which has become distinctive in relation to the goods kind, quality, quantity, intended purpose, value, geographical origin, time or production of
for which registration is requested as a result of the use that have been made of it in commerce the goods or rendering of the services, or other characteristics of the goods;
in the Philippines. The Office may accept as prima facie evidence that the mark has become (11) Consists of SHAPES that may be necessitated by technical factors or by the nature of the
distinctive, as used in connection with the applicant’s goods or services in commerce, proof of goods themselves or factors that affect their intrinsic value;
substantially exclusive and continuous use thereof by the applicant in commerce in the (12) Consists of COLOR alone, unless defined by a given form; or
Philippines for five (5) years before the date on which the claim of distinctiveness is made. (13) Is CONTRARY to public order or morality.

123.3. The nature of the goods to which the mark is applied will not constitute an obstacle to
registration. Doctrine of Secondary Meaning (Section 123.2)
While as a general rule, generic, indicative or descriptive marks ([j], [k], and [l] of Sec. 123.1) are
non-registrable, when such kind of mark has become distinctive, because of its long, continuous
and exclusive use for five (5) years, as used in connection with the applicant’s goods or services
What may not be registered? in commerce and in the mind of the public indicates a single source to consumers, it may be
(1) Consists of IMMORAL, deceptive or scandalous matter, or matter which may disparage or registered.
falsely suggest a connection with persons, living or dead, institutions, beliefs, or national
symbols, or bring them into contempt or disrepute;
(2) Consists of the FLAG or coat of arms or other insignia of the Philippines or any of its DEFINITION EXAMPLE
political subdivisions, or of any foreign nation, or any simulation thereof; Fanciful It is invented for the sole purpose of “Kodak” for camera and “Xerox”
(3) Consists of NAME, portrait or signature identifying a particular living individual except by marks functioning as a trademark and has no other for photocopying machine
his written consent, or the name, signature, or portrait of a deceased President of the meaning than acting as a mark.
Philippines, during the life of his widow, if any, except by written consent of the widow;
Arbitrary It is utilized as a device having a common “Adagio” as a musical instrument


!" DomingoŸAriolaŸMartinezŸMolaer

DAMM NOTES – LAW ON INTELLECTUAL PROPERTY SY $%&! - !"#!
Atty. Castillo-Taleon


marks meaning that has no relation to the goods or used to identify brasseries, or the 152.4. The use of a mark by a company related with the registrant or applicant shall inure to the
services being sold. word “Cosmopolite” for canned latter’s benefit, and such use shall not affect the validity of such mark or of its registration:
fish, or “Dutch Boy” for paints Provided, That such mark is not used in such manner as to deceive the public. If use of a mark
and paint color solutions by a person is controlled by the registrant or applicant with respect to the nature and quality of
the goods or services, such use shall inure to the benefit of the registrant or applicant.
Suggestive It suggests some attribute or benefit of “Airbus” for airplanes
marks the product or service with which they
are used, but they stop short of merely
describing the attributes of the product or
service.

Composite They are marks consisting of two or more “20th Century Nylon Shirts VI. TESTS TO DETERMINE CONFUSING SIMILARITY BETWEEN MARKS
marks elements or combination of words, phrases, Factory” must be required to
designs, symbols or color schemes. disclaim “nylon” and “shirts
Although they cannot be registered by factory DOMINANCY TEST HOLISTIC TEST
themselves, together they may be a part of
composite mark as long as they provide a
As to considerations
disclaimer. The person who registers them
Focuses on the similarity of the main, Mandates that the entirety of the marks in
as part of a mark will not acquire ownership
essential, dominant, or prevalent features of a question must be considered in determining
thereto.
mark. bar confusing similarity.
Coined Marks may be registered even if they are “Starbite” for metal polish,
As to basis of comparison
marks contradictions of or coined from generic and “Gaslam” for lamps, and
descriptive terms. “Mirrorlike” for furniture and floor Relies on visuals, aural, and connotative Relies only on visual comparison
polish comparisons and overall impressions between
the two trademarks.

As to test
The test is whether there is similarity of the The test is whether the general confusion
V. PRIOR USE OF MARK AS A REQUIREMENT prevalent features of the competing made by the articles upon the eye of the
trademarks which might cause confusion or casual purchaser who is unsuspicious and off
deception, and does thus constitute his guard, is such as to likely result in his
infringement. confounding it with the original.
Section 122. How Marks are Acquired. - The rights in a mark shall be acquired through
registration made validly in accordance with the provisions of this law. Note: Note:
Exact duplication or limitation is not Comparison of the words is not the only
necessary. determinant factor.
Section 152. Non-use of a Mark When Excused. -
152.1. Non-use of a mark may be excused if caused by circumstances arising independently of
the will of the trademark owner. Lack of funds shall not excuse non-use of a mark.
A. Dominancy Test
152.2. The use of the mark in a form different from the form in which it is registered, which does
not alter its distinctive character, shall not be ground for cancellation or removal of the mark and
Case No. 1
shall not diminish the protection granted to the mark.
ASIA BREWERY, INC. vs. THE HON. COURT OF APPEALS and SAN MIGUEL
CORPORATION
152.3. The use of a mark in connection with one or more of the goods or services belonging to
G.R. No. 103543
the class in respect of which the mark is registered shall prevent its cancellation or removal in
respect of all other goods or services of the same class. FACTS:
In September 1988, San Miguel Corporation (SMC) sued Asia Brewery Inc. for allegedly
infringing upon their trademark on their beer product popularly known as “San Miguel Pale

!" DomingoŸAriolaŸMartinezŸMolaer

DAMM NOTES – LAW ON INTELLECTUAL PROPERTY SY $%&! - !"#!
Atty. Castillo-Taleon


Pilsen”; that Asia Brewery’s “Beer na Beer” product, by infringing upon SMC’s trademark has and snack vans in Metro Manila and nearby provinces. Respondent corporation's menu includes
committed unfair competition as “Beer na Beer” creates confusion between the two products. hamburger sandwiches and other food items. On 21 October 1988, respondent corporation
The RTC ruled in favor of Asia Brewery but the Court of Appeals reversed the RTC. applied with the PBPTT for the registration of the "Big Mak" mark for its hamburger sandwiches,
which was opposed by McDonald's. McDonald's also informed LC Big Mak chairman of its
ISSUE: Whether Asia Brewery infringed upon the trademark of SMC. exclusive right to the "Big Mac" mark and requested him to desist from using the "Big Mac" mark
or any similar mark.
RULING:
NO. Both products are manufactured using amber colored steinie bottles of 320 ml. Both were Having received no reply, petitioners sued L.C. Big Mak Burger, Inc. and its directors before
labeled in a rectangular fashion using white color paint. But other than these similarities, there Makati RTC RTC, for trademark infringement and unfair competition. RTC rendered a Decision
are salient differences between the two. As found by the Supreme Court, among others they are finding respondent corporation liable for trademark infringement and unfair competition. CA
the following: reversed RTC's decision on appeal.
(1) The dominant feature of SMC’s trademark are the words “San Miguel Pale Pilsen” while
that of Asia Brewery’s trademark is the word “Beer”. Nowhere in SMC’s product can be ISSUE: Whether respondent corporation is liable for trademark infringement and unfair
seen the word “Beer” nor in Asia Brewery’s product can be seen the words “San Miguel competition
Pale Pilsen”. Surely, someone buying “Beer na Beer” cannot mistake it as “San Miguel
Pale Pilsen” beer. RULING:
(2) The bottle designs are different. SMC’s bottles have slender tapered neck while that of YES. Section 22 of Republic Act No. 166, as amended, defines trademark infringement as
“Beer na Beer” are fat. Though both beer products use steinie bottles, SMC cannot claim follows:
that Asia Brewery copied the idea from SMC. SMC did not invent but merely borrowed the “Infringement, what constitutes. - Any person who [1] shall use, without the consent of the
steinie bottle from abroad and SMC does not have any patent or trademark to protect the registrant, any reproduction, counterfeit, copy or colorable imitation of any registered mark or
steinie bottle shape and design. trade-name in connection with the sale, offering for sale, or advertising of any goods, business
(3) In SMC bottles, the words “pale pilsen” are written diagonally while in “Beer na Beer”, the or services on or in connection with which such use is likely to cause confusion or mistake or to
words “pale pilsen” are written horizontally. Further, the words “pale pilsen” cannot be said deceive purchasers or others as to the source or origin of such goods or services, or identity of
to be copied from SMC for “pale pilsen” are generic words which originated from Pilsen, such business; or [2] reproduce, counterfeit, copy, or colorably imitate any such mark or trade-
Czechoslovakia. “Pilsen” is a geographically descriptive word and is non-registrable. name and apply such reproduction, counterfeit, copy, or colorable imitation to labels, signs,
(4) SMC bottles have no slogans written on them while Asia Brewery’s bottles have a prints, packages, wrappers, receptacles or advertisements intended to be used upon or in
copyrighted slogan written on them that is “Beer na Beer”. connection with such goods, business or services, shall be liable to a civil action by the
(5) In SMC bottles, it is expressly labeled as manufactured by SMC. In Asia Brewery beer registrant for any or all of the remedies herein provided.”
products, it is likewise expressly labeled as manufactured by Asia Brewery. Surely, there
is no intention on the part of Asia Brewery to confuse the public and make it appear that To establish trademark infringement, the following elements must be shown:
“Beer na Beer” is a product of SMC, a long-established and more popular brand. (1) The validity of plaintiff's mark;
(2) The plaintiff's ownership of the mark; and
(3) The use of the mark or its colorable imitation by the alleged infringer results in "likelihood of
confusion."
Case No. 2
MCDONALD'S CORPORATION and MCGEORGE FOOD INDUSTRIES, INC., vs. Of these, it is the element of likelihood of confusion that is the gravamen of trademark
L.C. BIG MAK BURGER, INC., FRANCIS B. DY, EDNA A. DY, RENE B. DY, WILLIAM B. DY, infringement.
JESUS AYCARDO, ARACELI AYCARDO, and GRACE HUERTO
G.R. No. 143993 1st element:
A mark is valid if it is distinctive and not merely generic and descriptive. The "Big Mac" mark,
FACTS: which should be treated in its entirety and not dissected word for word, is neither generic nor
Petitioner McDonald's Corporation ("McDonald's") is a US corporation that operates a global descriptive. Generic marks are commonly used as the name or description of a kind of goods,
chain of fast-food restaurants, with Petitioner McGeorge Food Industries ("McGeorge"), as the such as "Lite" for beer. Descriptive marks, on the other hand, convey the characteristics,
Philippine franchisee. McDonald's owns the "Big Mac" mark for its "double-decker hamburger functions, qualities or ingredients of a product to one who has never seen it or does not know it
sandwich." with the US Trademark Registry on 16 October 1979. Based on this Home exists, such as "Arthriticare" for arthritis medication. On the contrary, "Big Mac" falls under the
Registration, McDonald's applied for the registration of the same mark in the Principal Register class of fanciful or arbitrary marks as it bears no logical relation to the actual characteristics of
of the then Philippine Bureau of Patents, Trademarks and Technology ("PBPTT") (now IPO). On the product it represents. As such, it is highly distinctive and thus valid.
18 July 1985, the PBPTT allowed registration of the "Big Mac."
2nd element:
Respondent L.C. Big Mak Burger, Inc. is a domestic corporation which operates fast-food outlets

!" DomingoŸAriolaŸMartinezŸMolaer

DAMM NOTES – LAW ON INTELLECTUAL PROPERTY SY $%&! - !"#!
Atty. Castillo-Taleon


Petitioners have duly established McDonald's exclusive ownership of the "Big Mac" mark. Prior been using Robert Jaworski and Ric Puno Jr. as their commercial advertisers; and that in those
valid registrants of the said mark had already assigned his rights to McDonald's. commercials Jaworski is a master of basketball and that Puno is a master of talk shows; that the
brand of coffee equitable or fit to them is Master Blend and Master Roast. CFC Corporation on
3rd element: the other hand alleged that the word “Master” is a generic and a descriptive term, hence not
Section 22 covers two types of confusion arising from the use of similar or colorable imitation subject to trademark. The Director of Patents ruled in favor of Nestle but the Court of Appeals,
marks, namely, confusion of goods (confusion in which the ordinarily prudent purchaser would using the Holistic Test, reversed the said decision.
be induced to purchase one product in the belief that he was purchasing the other) and
confusion of business (though the goods of the parties are different, the defendant's product is ISSUE: Whether the Court of Appeals is correct
such as might reasonably be assumed to originate with the plaintiff, and the public would then be
deceived either into that belief or into the belief that there is some connection between the RULING:
plaintiff and defendant which, in fact, does not exist). NO. The proper test that should have been used is the Dominancy Test. The application of the
totality or holistic test is improper since the ordinary purchaser would not be inclined to notice the
There is confusion of goods in this case since respondents used the "Big Mak" mark on the specific features, similarities or dissimilarities, considering that the product is an inexpensive and
same goods, i.e. hamburger sandwiches, that petitioners' "Big Mac" mark is used. common household item. The use of the word Master by Nestle in its products and commercials
has made Nestle acquire a connotation that if it’s a Master product it is a Nestle product. As
There is also confusion of business due to Respondents' use of the "Big Mak" mark in the sale such, the use by CFC of the term “MASTER” in the trademark for its coffee product FLAVOR
of hamburgers, the same business that petitioners are engaged in, also results in confusion of MASTER is likely to cause confusion or mistake or even to deceive the ordinary purchasers.
business. The registered trademark owner may use his mark on the same or similar products, in
different segments of the market, and at different price levels depending on variations of the In addition, the word “MASTER” is neither a generic nor a descriptive term. As such, said term
products for specific segments of the market. The registered trademark owner enjoys protection cannot be invalidated as a trademark and, therefore, may be legally protected.
in product and market areas that are the normal potential expansion of his business.
Generic terms are those which constitute “the common descriptive name of an article or
Furthermore, In determining likelihood of confusion, the SC has relied on the dominancy test substance,” or comprise the “genus of which the particular product is a species,” or are
(similarity of the prevalent features of the competing trademarks that might cause confusion) “commonly used as the name or description of a kind of goods,” or “imply reference to every
over the holistic test (consideration of the entirety of the marks as applied to the products, member of a genus and the exclusion of individuating characters,” or “refer to the basic nature of
including the labels and packaging). the wares or services provided rather than to the more idiosyncratic characteristics of a
particular product,” and are not legally protectable.
Applying the dominancy test, Respondents' use of the "Big Mak" mark results in likelihood of
confusion. Aurally the two marks are the same, with the first word of both marks phonetically the On the other hand, a term is descriptive and therefore invalid as a trademark if, as understood in
same, and the second word of both marks also phonetically the same. Visually, the two marks its normal and natural sense, it “forthwith conveys the characteristics, functions, qualities or
have both two words and six letters, with the first word of both marks having the same letters ingredients of a product to one who has never seen it and does not know what it is,” or “if it
and the second word having the same first two letters. forthwith conveys an immediate idea of the ingredients, qualities or characteristics of the goods,”
or if it clearly denotes what goods or services are provided in such a way that the consumer
Lastly, since Section 22 only requires the less stringent standard of "likelihood of confusion," does not have to exercise powers of perception or imagination.
Petitioners' failure to present proof of actual confusion does not negate their claim of trademark
infringement. Rather, the term “MASTER” is a suggestive term brought about by the advertising scheme of
Nestle. Suggestive terms are those which, in the phraseology of one court, require “imagination,
thought and perception to reach a conclusion as to the nature of the goods.” Such terms, “which
subtly connote something about the product,” are eligible for protection in the absence of
Case No. 3 secondary meaning. While suggestive marks are capable of shedding “some light” upon certain
SOCIETE DES PRODUITS NESTLE, S.A. and NESTLE PHILIPPINES, INC. vs. characteristics of the goods or services in dispute, they nevertheless involve “an element of
COURT OF APPEALS and CFC CORPORATION incongruity,” “figurativeness,” or ”imaginative effort on the part of the observer.”

FACTS:
In 1984, CFC Corporation filed with the Bureau of Patents, Trademarks, and Technology
Transfers an application for the registration of its trademark “Flavor Master” – an instant coffee.
Nestle opposed the application as it alleged that “Flavor Master” is confusingly similar to Nestle
coffee products like Master Blend and Master Roast. Nestle alleged that in promoting their
products, the word Master has been used so frequently so much so that when one hears the
word Master it connotes to a Nestle product. They provided as examples the fact that they’ve

!" DomingoŸAriolaŸMartinezŸMolaer

DAMM NOTES – LAW ON INTELLECTUAL PROPERTY SY $%&! - !"#!
Atty. Castillo-Taleon


B. Holistic Test factors, we conclude that the similarities in the trademarks in question are not sufficient as to
likely cause deception and confusion tantamount to infringement.
Case No. 1
EMERALD GARMENT MANUFACTURING CORPORATION vs. HON. COURT OF APPEALS, Further, H.D. Lee failed to prove in court that it had prior actual commercial use of its “LEE”
BUREAU OF PATENTS, TRADEMARKS AND TECHNOLOGY TRANSFER and H.D. LEE trademark in the Philippines. H.D. Lee did show certificates of registrations for its brand but
COMPANY, INC. registration is not sufficient. Actual use in commerce in the Philippines is an essential
G.R. No. 100098 prerequisite for the acquisition of ownership over a trademark pursuant to Sec. 2 and 2-A of the
Philippine Trademark Law (R.A. No. 166).
FACTS: A rule widely accepted and firmly entrenched because it has come down through the years is
In 1981, H.D Lee Co., Inc., a foreign company, filed an opposition against the trademark that actual use in commerce or business is a prerequisite in the acquisition of the right of
application of Emerald Garment. Allegedly, the trademark “Stylistic Mr. Lee” sought to be applied ownership over a trademark.
for by Emerald Garment is too confusingly similar with the brand “Lee” which has for its
variations “Lee Riders”, “Lee Sures”, and “Lee Leens”. The Director of Patents as well as the It would seem quite clear that adoption alone of a trademark would not give exclusive right
Court of Appeals ruled in favor of H.D. Lee Co. thereto. Such right “grows out of their actual use.” Adoption is not use. One may make
advertisements, issue circulars, give out price lists on certain goods; but these alone would not
ISSUE: Whether the decision of the Court of Appeals is correct give exclusive right of use. For trademark is a creation of use. The underlying reason for all
these is that purchasers have come to understand the mark as indicating the origin of the wares.
RULING: Flowing from this is the trader’s right to protection in the trade he has built up and the goodwill
NO. The Supreme Court considered that the trademarks involved as a whole and ruled that he has accumulated from use of the trademark. Registration of a trademark, of course, has
Emerald Garment’s “STYLISTIC MR. LEE” is not confusingly similar to H.D. Lee’s “LEE” value: it is an administrative act declaratory of a pre-existing right. Registration does not,
trademark. The trademark “Stylistic Mr. Lee”, although on its label the word “LEE” is prominent, however, perfect a trademark right.
the trademark should be considered as a whole and not piecemeal. The dissimilarities between
the two marks become conspicuous, noticeable and substantial enough to matter especially in
the light of the following variables that must be factored in.
Case No. 2
First, the products involved in the case at bar are, in the main, various kinds of jeans. These are BRISTOL MYERS COMPANY vs. THE DIRECTOR OF PATENTS and UNITED AMERICAN
not your ordinary household items like catsup, soysauce or soap which are of minimal cost. PHARMACEUTICALS, INC.
Maong pants or jeans are not inexpensive. Accordingly, the casual buyer is predisposed to be G.R. No. L-21587
more cautious and discriminating in and would prefer to mull over his purchase. Confusion and
deception, then, is less likely. FACTS:
A petition for registration in the Principal Register of the Patent Office of the trademark
Second, like his beer, the average Filipino consumer generally buys his jeans by brand. He does "BIOFERIN" was filed on October 21, 1957 by United American Pharmaceuticals, Inc. Said
not ask the sales clerk for generic jeans but for, say, a Levis, Guess, Wrangler or even an domestic corporation first used the afore-stated trademark in the Philippines on August 13, 1957.
Armani. He is, therefore, more or less knowledgeable and familiar with his preference and will It covers "a medicinal preparation of antihistamic, analgesic, antipyritic with vitamin C and
not easily be distracted. Bioflavenoid used in the treatment of common colds, influenza and other febrile diseases with
capillary hemmorrhagic tendencies." The product falls under Class 6 of the official classification,
Finally, in line with the foregoing discussions, more credit should be given to the “ordinary that is, "Medicines and Pharmaceutical Preparations".
purchaser.” Cast in this particular controversy, the ordinary purchaser is not the “completely
unwary consumer” but is the “ordinarily intelligent buyer” considering the type of product Bristol Myers Co., a corporation of the State of Delaware, U.S.A., filed on January 6, 1959 an
involved. opposition to the application. Said oppositor is the owner in the Philippines of the trademark
"BUFFERIN" under Certificate of Registration No. 4578 issued by the Philippine Patent Office on
There is no cause for the Court of Appeal’s apprehension that Emerald Garment’s products March 3, 1954. Its trademark is also registered in the United States under Certificate of
might be mistaken as “another variation or line of garments under H.D. Lee’s ‘LEE’ Registration No. 566190 issued on November 4, 1952. It was first used in the Philippines on May
trademark”. As one would readily observe, H.D. Lee’s variation follows a standard format 13, 1953. The product covered by "BUFFERIN" also belongs to Class 6, Medicines and
“LEERIDERS,” “LEESURES” and “LEELEENS.” It is, therefore, improbable that the public would Pharmaceutical Preparations. Designated as "Antacid analgesic", it is intended for relief in cases
immediately and naturally conclude that petitioner’s “STYLISTIC MR. LEE” is but another of "simple headaches, neuralgia, colds, menstrual pain and minor muscular aches."
variation under H.D. Lee’s “LEE” mark.
The Director of Patents rendered a decision granting the petition for registration and dismissing
The issue of confusing similarity between trademarks is resolved by considering the distinct the opposition, on the ground that, all factors considered the trademarks in question are not
characteristics of each case. In the present controversy, taking into account these unique confusingly similar, so that the damage feared by the oppositor will not result.

!" DomingoŸAriolaŸMartinezŸMolaer

DAMM NOTES – LAW ON INTELLECTUAL PROPERTY SY $%&! - !"#!
Atty. Castillo-Taleon


ISSUE:
Whether the trademarks "BIOFERIN" and "BUFFERIN", as presented to the public in their CRITERIA
respective labels, confusingly similar A mark is well-known according to Sec. 123.1(e) if it is considered by the competent authority of
the Philippines to be well-known internationally and in the Philippines and that in determining
RULING: whether a mark is well-known, account shall be taken of the knowledge of the relevant sector of
NO. "BIOFERIN" and "BUFFERIN", as presented to the public in their respective labels, are not the public, rather than of the public at large, including knowledge in the Philippines which has
confusingly similar. In determining whether two trademarks are confusingly similar, the test is been obtained as a result of the promotion of the mark. Competent authority as used in this
not simply to take their words and compare the spelling and pronunciation of said words. Rather, section includes the IPO and its relevant officers, the DTI and of course, the SC, as the final
it is to consider the two marks in their entirety, as they appear in the respective labels, in relation arbiter in this country.
to the goods to which they are attached.
Rule 102 of the Rules and Regulations on Trademarks, Service Marks, Trade Names and
Applying this test to the trademarks involved in this case, it is at once evident that the Director of Marked and Stamped Containers provides help in determining if a mark is well-known.
Patents did not err in finding no confusing similarity. For though the words "BIOFERIN" and
"BUFFERIN" have the same suffix and similar sounding prefixes, they appear in their respective
labels with strikingly different backgrounds and surroundings, as to color , size and design.
In determining whether a mark is well-known, the following criteria or any combination thereof
Accordingly, taken as they will appear to a prospective customer, the trademark in question are may be taken into account:
not apt to confuse. Furthermore, the product of the applicant is expressly stated as dispensable (a) The duration, extent and geographical area of any use of the mark, in particular, the
only upon doctor's prescription, while that of oppositor does not require the same. The chances duration, extent and geographical area of any promotion of the mark, including advertising
of being confused into purchasing one for the other are therefore all the more rendered or publicity and the presentation, at fairs or exhibitions, of the goods and/or services to
negligible. Although oppositor avers that some drugstores sell "BIOFERIN" without asking for a which the mark applies;
doctor's prescription, the same if true would be an irregularity not attributable to the applicant, (b) The market share, in the Philippines and in other countries, of the goods and/or services to
who has already clearly stated the requirement of a doctor's prescription upon the face of the which the mark is applied;
label of its product. (c) The degree of the inherent or acquired distinction of the mark;
(d) The quality-image or reputation acquired by the mark;
(e) The extent to which the mark has been registered in the world;
(f) The exclusivity of registration attained by the mark in the world;
VII. WELL-KNOWN MARKS (g) The extent to which the mark has been used in the world;
(h) The exclusivity of use attained by the mark in the world;
(i) The commercial value attributed to the mark in the world;
Section 123. Registrability. - (j) The record of successful protection of the rights in the mark;
123.1. A mark cannot be registered if it: (k) The outcome of litigations dealing with the issue of whether the mark is a well-known mark;
(e) Is identical with, or confusingly similar to, or constitutes a translation of a mark which is and,
considered by the competent authority of the Philippines to be well-known internationally (l) The presence or absence of identical or similar marks validly registered for or used on
and in the Philippines, whether or not it is registered here, as being already the mark of a identical or similar goods or services and owned by persons other than the person claiming
person other than the applicant for registration, and used for identical or similar goods or that his mark is a well-known mark.
services: Provided, That in determining whether a mark is well-known, account shall be
taken of the knowledge of the relevant sector of the public, rather than of the public at
large, including knowledge in the Philippines which has been obtained as a result of the
promotion of the mark; VIII. RIGHTS CONFERRED BY REGISTRATION

(f) Is identical with, or confusingly similar to, or constitutes a translation of a mark considered
well-known in accordance with the preceding paragraph, which is registered in the Section 147. Rights Conferred. -
Philippines with respect to goods or services which are not similar to those with respect to 147.1. The owner of a registered mark shall have the exclusive right to prevent all third parties
which registration is applied for: Provided, That use of the mark in relation to those goods not having the owner's consent from using in the course of trade identical or similar signs or
or services would indicate a connection between those goods or services, and the owner of containers for goods or services which are identical or similar to those in respect of which the
the registered mark: Provided further, That the interests of the owner of the registered mark trademark is registered where such use would result in a likelihood of confusion. In case of the
are likely to be damaged by such use;


!" DomingoŸAriolaŸMartinezŸMolaer

DAMM NOTES – LAW ON INTELLECTUAL PROPERTY SY $%&! - !"#!
Atty. Castillo-Taleon


use of an identical sign for identical goods or services, a likelihood of confusion shall be IX. USE BY THIRD PARTIES OF NAMES, ETC SIMILAR TO REGISTERED MARK
presumed.

147.2. The exclusive right of the owner of a well-known mark defined in Subsection 123.1(e) Section 148. Use of Indications by Third Parties for Purposes Other than those for which
which is registered in the Philippines, shall extend to goods and services which are not similar to the Mark is Used. - Registration of the mark shall not confer on the registered owner the right to
those in respect of which the mark is registered: Provided, That use of that mark in relation to preclude third parties from using bona fide their names, addresses, pseudonyms, a geographical
those goods or services would indicate a connection between those goods or services and the name, or exact indications concerning the kind, quality, quantity, destination, value, place of
owner of the registered mark: Provided further, That the interests of the owner of the registered origin, or time of production or of supply, of their goods or services: Provided, That such use is
mark are likely to be damaged by such use. (n) confined to the purposes of mere identification or information and cannot mislead the public as
to the source of the goods or services. (n)

X. INFRINGEMENT AND REMEDIES

A. Trademark Infringement
Registered Marks
General Rule: There is exclusive right to prevent third persons from using identical or similar
signs or containers for identical or similar goods or services, where such use would result in a
Section 155. Remedies; Infringement. - Any person who shall, without the consent of the
likelihood of confusion.
owner of the registered mark:
• In case of the use of an identical sign for identical or similar goods or services, a likelihood of
155.1. Use in commerce any reproduction, counterfeit, copy, or colorable imitation of a
confusion shall be presumed.
registered mark or the same container or a dominant feature thereof in connection with the sale,
• Exception: In cases of importation of drugs and medicines allowed under Sec. 72.1 and of
offering for sale, distribution, advertising of any goods or services including other preparatory
off-patent drugs and medicines, provided that said drugs or medicines bear the registered steps necessary to carry out the sale of any goods or services on or in connection with which
marks that have not been tampered, unlawfully modified, or infringed upon. such use is likely to cause confusion, or to cause mistake, or to deceive; or

155.2. Reproduce, counterfeit, copy or colorably imitate a registered mark or a dominant feature
Registered Well-Known Mark thereof and apply such reproduction, counterfeit, copy or colorable imitation to labels, signs,
Exclusive right to prevent third persons from using identical or similar signs even for dissimilar or
prints, packages, wrappers, receptacles or advertisements intended to be used in commerce
unrelated goods or services, provided that the use will indicate a connection between the goods upon or in connection with the sale, offering for sale, distribution, or advertising of goods or
and the owner of the mark and that the interest of the owner would likely be damaged. services on or in connection with which such use is likely to cause confusion, or to cause
mistake, or to deceive, shall be liable in a civil action for infringement by the registrant for the
remedies hereinafter set forth: Provided, That the infringement takes place at the moment any of
Doctrine of Related Goods or Services the acts stated in Subsection 155.1 or this subsection are committed regardless of whether there
There is infringement when there is use of similar marks on goods that are so related that the
is actual sale of goods or services using the infringing material. (Sec. 22, R.A. No 166a)
public may be, or is actually deceived, and misled that the goods came from the same maker or
manufacturer (Esso Standard Eastern Inc. vs CA, GR No. 29971)

An exception to this doctrine is the additional right granted to a registered well-known mark.
Mere unauthorized use of a container bearing a registered trademark in connection with the
sale, distribution or advertising of gods or services which is likely to cause confusion, mistake or
Doctrine of Dilution deception among the buyers/consumers can be considered as trademark infringement.
Copying which, while not sufficiently confusing to divert sales in the short run, will tend to divert
them in the long run by weakening the instantaneous favorable associations the public makes
with highly regarded products. There are two types of confusion in trademark infringement:
CONFUSION OF GOODS CONFUSION OF BUSINESS
When an otherwise prudent purchaser is When the goods of the parties are different but
induced to purchase one product in the belief the defendant’s product can reasonably
that he is purchasing another, in which case (though mistakenly) be assumed to originate
defendant’s goods are then bought as the form the plaintiff, thus deceiving the public into
plaintiff’s and its poor quality reflects badly on believing that there is some connection


!" DomingoŸAriolaŸMartinezŸMolaer

DAMM NOTES – LAW ON INTELLECTUAL PROPERTY SY $%&! - !"#!
Atty. Castillo-Taleon


the plaintiff’s reputation between the plaintiff and defendant which, in 157.1. In any action arising under this Act, in which a violation of any right of the owner of the
fact, does not exist registered mark is established, the court may order that goods found to be infringing be, without
compensation of any sort, disposed of outside the channels of commerce in such a manner as to
avoid any harm caused to the right holder, or destroyed; and all labels, signs, prints, packages,
wrappers, receptacles and advertisements in the possession of the defendant, bearing the
PARALLEL IMPORTATION refers to goods produced and sold legally, and subsequently registered mark or trade name or any reproduction, counterfeit, copy or colorable imitation
exported. thereof, all plates, molds, matrices and other means of making the same, shall be delivered up
• In the importing country, such goods may create havoc particularly for entrepreneurs who and destroyed.
sell the same goods, obtained via different distribution channels and perhaps more
expensively. 157.2. In regard to counterfeit goods, the simple removal of the trademark affixed shall not be
• In order to exclude such unwelcome competition, intellectual property rights have sometimes sufficient other than in exceptional cases which shall be determined by the Regulations, to
been of help. If products sold or imported by third parties fall within the scope of patents, permit the release of the goods into the channels of commerce. (Sec. 24, R.A. No. 166a)
trademarks or copyrights valid in this particular country, such sale or importation by third
parties is generally deemed infringing. Owners of products covered by intellectual property
rights have the exclusive right to put such products on the market. On the other hand, there
is little doubt that once the owner of an intellectual property right has put such goods on the Civil vs Criminal Infringement
market either himself or with his consent, there is little he can do about further acts of
commercial exploitation, such as re-sale, etc., on the domestic market.
• Even if a car is covered by a number of patents, once the car maker has put that car on the
market, there is a consensus that he cannot prevent that car from being re-sold, leased-out, C. Jurisdiction
etc.
[A.M. No. 02-1-11-SC February 19, 2002]
RE: DESIGNATION OF AN INTELLECTUAL PROPERTY JUDGE FOR MLA.

B. Damages EN BANC

Gentlemen:
Section 156. Actions, and Damages and Injunction for Infringement. - Quoted hereunder, for your information, is a resolution of this Court dated FEB 19 2002.
156.1. The owner of a registered mark may recover damages from any person who infringes his
rights, and the measure of the damages suffered shall be either the reasonable profit which the A.M. No. 02-1-11-SC (Re:Designation of an Intellectual Property Judge for Manila.)
complaining party would have made, had the defendant not infringed his rights, or the profit (A) Acting on the memorandum dated 15 January 2002 of Deputy Court Administrator
which the defendant actually made out of the infringement, or in the event such measure of Christopher O. Lock, favorably endorsed by Court Administrator Presbitero J. Velasco, Jr.,
damages cannot be readily ascertained with reasonable certainty, then the court may award as the Court hereby designates Branch 24 of the Regional Trial Court of Manila, presided over
damages a reasonable percentage based upon the amount of gross sales of the defendant or by Judge ANTONIO M. EUGENIO, JR., as Special Intellectual Property Court for Manila in
the value of the services in connection with which the mark or trade name was used in the substitution of Branch 1 of said Court which was designated Special Intellectual Property
infringement of the rights of the complaining party. (Sec. 23, first par., R.A. No. 166a) Court, then presided over by Judge Rebecca de Guia Salvador (now Associate Justice of
the Court of Appeals), pursuant to Administrative Order No. 113-95 dated 5 October 1995.
156.2. On application of the complainant, the court may impound during the pendency of the
action, sales invoices and other documents evidencing sales. (n) As judge of the Special Intellectual Property Court, Judge Antonio M. Eugenio, Jr. shall try
and decide cases involving violations of intellectual property rights under the Intellectual
156.3. In cases where actual intent to mislead the public or to defraud the complainant is shown, Property Code (R.A. No. 8293) committed within the City of Manila.
in the discretion of the court, the damages may be doubled. (Sec. 23, first par., R.A. No. 166)
The earlier designation of Branch 1 of the Regional Trial Court of Manila as a Special
156.4. The complainant, upon proper showing, may also be granted injunction. (Sec. 23, second Intellectual Property Court is hereby REVOKED, and the cases involving violations of
par., R.A. No. 166a) intellectual property rights earlier assigned to said Branch 1 pursuant to and by virtue of
Administrative Order No. 113-95 are hereby re-assigned to Branch 24 of said Court.

Section 157. Power of Court to Order Infringing Material Destroyed. - (B) Furthermore, acting on the recommendation of Hon. Associate Justice Reynato S. Puno,
Chairman of the Committee on Revision of the Rules of Court, and the Office of the Court

!" DomingoŸAriolaŸMartinezŸMolaer

DAMM NOTES – LAW ON INTELLECTUAL PROPERTY SY $%&! - !"#!
Atty. Castillo-Taleon


Administrator, and in order to ensure speedy disposition of cases involving violations of by the branches to which they have been assigned. Only cases hereafter filed may be
intellectual property rights under the Intellectual Property Code (R.A. No. 8293), the assigned to the above designated special courts.
following Regional Trial Courts (RTCs) are hereby designated Special Intellectual Property
Courts: (C) Finally, in order to ensure a just and equitable distribution of cases among the Judges
concerned, all the aforementioned Special Intellectual Property Courts shall continue to
participate in the raffles of other cases: Provided, however, that the Executive Judges
Region 1: concerned shall adopt a procedure whereby every intellectual property right case assigned
1)Hon. Antonio M. Esteves to a Special Intellectual Property Court should be considered a case raffled to it and be duly
Presiding Judge, Branch 5 credited to such court.
RTC, Baguio City
This Resolution shall take effect immediately and the Office of the Court Administrator shall
2)Hon. Manuel L. Argel implement it.
Presiding Judge, Branch 65
RTC, Laoag City

[A.M. No. 03-03-03 SC July 1, 2003]


Region 2: RE: CONSOLIDATION OF INTELLECTUAL PROPERTY COURTS WITH COMMERCIAL
1)Hon. Rolando R. Velasco COURTS
Presiding Judge, Branch 6 WHEREAS, to implement the provisions of Section 5.2 of Republic Act No. 8799 (The Securities
RTC, Aparri, Cagayan Regulation Code), and in the interest of a speedy and efficient administration of justice, the
Supreme Court en bane, in the (a) Resolution dated 21 November 2000 (Annex 1), 4 July 2001
(Annex 1-a), 12 November 2002 (Annex 1-b), and 9 July 2002 (Annex 1-c), all issued in A.M.
Region 5: No. 00-11-03-SC; (b) Resolution dated 27 August 2001 in A.M. No. 01-5-298 RTC (Annex 2);
1)Hon. Vladimir B. Bruselas and (c) Resolution dated 8 July 2002 in A.M. No. 01-12-656-RTC (Annex 3), resolved to
Presiding Judge, Branch 6 designate certain branches of the Regional Trial Courts to try and decide cases formerly
RTC, Legazpi City cognizable by the Securities and Exchange Commission;

2)Hon. Filemon B. Montenegro WHEREAS, pursuant to the same Resolution, sixty-five (65) Regional Trial Courts, distributed in
Presiding Judge, Branch 26 all regions (NCJR and Regions 1-12), were designated as SEC courts ("SEC Courts"), which
RTC, Naga City courts have presently a total of 812 pending SEC cases (see Annex 6, Table);

WHEREAS, in A.O No. 113-95, dated 2 October 1995, as amended by A.O. No. 104-96, dated
Region 8: 21 October 1996, the Regional Trial Courts in the National Capital Region and Regions 3, 4, 6,
1)Hon. Frisco T. Lilagan 7, 9, 10 and 11, with twenty-seven (27) judges, were specially designated to try and decide
Presiding Judge, Branch 34 cases for violations of Intellectual Property Rights (Annex 4), and to ensure the speedy
RTC, Tacloban City disposition of cases involving violations of intellectual property rights under the Intellectual
Property Code (Rep. Act No. 8293), the Supreme Court en bane, in A.M. No. 02-1-11- SC, dated
February 19, 2002, designated the Regional Trial Courts in Regions 1, 2, 5, 8 and 12, with a
Region 12: total of seven (7) judges, and Branch 24 of the Regional Trial Court of Manila with one (1) judge,
1)Hon. Albert B. Abragan as Special Intellectual Property Courts ("Special IP Courts") (Annex 5)
Presiding Judge, Branch 3
RTC, Iligan City WHEREAS, pursuant to A.M. No. 02-1-11 SC and A.O. No. 113-95, these Special IP Courts
have a total caseload of 503 cases. Of this number 434 IP cases are pending in the NCJR
(Annex 6, Table);
The foregoing Special Intellectual Property Courts shall try and decide cases involving
violations of intellectual property rights defined under the Intellectual Property Code WHEREAS, since the establishment of Special IP Courts (except for the Special IP Courts in
committed within their respective territorial areas. Since there are only a few cases of Manila), 15 designated courts, in Regions 1, 2, 3, 4, 5, 6, 7, 8, 9 and 12 have zero (0) IP cases,
violations of intellectual property rights now pending in other branches of the and do not warrant their continued designations as Intellectual Property Courts (Annex 7, Table);
aforementioned Regional Trial Courts, such cases shall remain with and shall be decided
WHEREAS, intellectual property cases are commercial in nature;

!! DomingoŸAriolaŸMartinezŸMolaer

DAMM NOTES – LAW ON INTELLECTUAL PROPERTY SY $%&! - !"#!
Atty. Castillo-Taleon


total damages claimed are not less than Two hundred thousand pesos (P200,000):
WHEREAS, to streamline the court structure and to promote expediency and efficiency in Provided further, That availment of the provisional remedies may be granted in accordance
handling such special cases, the jurisdiction to hear and decide IPC and SEC cases are best with the Rules of Court. The Director of Legal Affairs shall have the power to hold and
consolidated in one court; punish for contempt all those who disregard orders or writs issued in the course of the
proceedings. (n)
NOW, THEREFORE, the Court Resolves:
1. The Regional Courts previously designated as SEC Courts through the: (a) Resolutions of (b) After formal investigation, the Director for Legal Affairs may impose one (1) or more of the
this Court dated 21 November 2000, 4 July 2001, 12 November 2002, and 9 July 2002, all following administrative penalties:
issued in A.M. No. 00-11-03-SC, (b) Resolution dated 27 August 2001 in A.M. No. 01-5- (i) The issuance of a cease and desist order which shall specify the acts that the
298-RTC; and (c) Resolution dated 8 July 2002 in A.M. No. 01-12-656-RTC are hereby respondent shall cease and desist from and shall require him to submit a
DESIGNATED and shall be CALLED as Special Commercial Courts to try and decide compliance report within a reasonable time which shall be fixed in the order;
cases involving violations of Intellectual Property Rights which fall within their jurisdiction (ii) The acceptance of a voluntary assurance of compliance or discontinuance as may
and those cases formerly cognizable by the Securities and Exchange Commission; be imposed. Such voluntary assurance may include one or more of the following:
2. The designation of Intellectual Property Courts under Administrative Order No. 113-95 1. An assurance to comply with the provisions of the intellectual property law
dated 2 October 1995, as amended by Administrative Order No. 104-96 dated 21 October violated;
1996 and Resolution dated 19 February 2002 in A.M. No. 02-1-11-SC, is hereby revoked. 2. An assurance to refrain from engaging in unlawful and unfair acts and practices
However, the Regional Trial Court, Branch 24, Manila is hereby designated as an additional subject of the formal investigation;
Special Commercial Court in the City of Manila; 3. An assurance to recall, replace, repair, or refund the money value of defective
3. Upon the effectivity of this Resolution, all IP cases shall be transferred to the designated goods distributed in commerce; and
Special Commercial Courts except those which have undergone the pretrial stage in civil 4. An assurance to reimburse the complainant the expenses and costs incurred in
cases or those where any of the accused has been arraigned in criminal cases which shall prosecuting the case in the Bureau of Legal Affairs. The Director of Legal Affairs
be retained by the court previously assigned to try them; may also require the respondent to submit periodic compliance reports and file
4. The Special Commercial Courts shall have jurisdiction over cases arising within their a bond to guarantee compliance of his undertaking;
respective territorial jurisdiction with respect to the National Capital Judicial Region and (iii) The condemnation or seizure of products which are subject of the offense. The
within the respective provinces with respect to the First to Twelfth Judicial Regions. Thus, goods seized hereunder shall be disposed of in such manner as may be deemed
cases shall be filed in the Office of the Clerk of Court in the official station of the designated appropriate by the Director of Legal Affairs, such as by sale, donation to distressed
Special Commercial Court. In the event of inhibition of the judge of a designated Special local governments or to charitable or relief institutions, exportation, recycling into
Commercial Court, the following guidelines shall be observed: (a) where there is only one other goods, or any combination thereof, under such guidelines as he may provide;
(1) Special Commercial Court, the case shall be raffled among the other judges in the (iv) The forfeiture of paraphernalia and all real and personal properties which have
station; (b) where there are two (2) Special Commercial Courts in the station, the Executive been used in the commission of the offense;
Judge shall immediately assign the case to the other Special Commercial Court; and (c) in (v) The imposition of administrative fines in such amount as deemed reasonable by the
case of inhibition of both judges of the Special Commercial Courts, the Executive Judge Director of Legal Affairs, which shall in no case be less than Five thousand pesos
shall raffle the case among the judges in the station; and (P5,000) nor more than One hundred fifty thousand pesos (P150,000). In addition,
5. In order to ensure a just and equitable distribution of cases, the designated Special an additional fine of not more than One thousand pesos (P1,000) shall be imposed
Commercial Courts shall continue to participate in the raffles of other cases. Provided, for each day of continuing violation;
however, that the Executive Judge concerned shall adopt a procedure whereby every IP (vi) The cancellation of any permit, license, authority, or registration which may have
and SEC case assigned to a Special Commercial Court should be considered a case been granted by the Office, or the suspension of the validity thereof for such period
raffled to it and duly credited to such court. of time as the Director of Legal Affairs may deem reasonable which shall not
exceed one (1) year;
This Resolution shall take effect on 1 July 2003 and shall be published in two (2) newspapers of (vii) The withholding of any permit, license, authority, or registration which is being
general circulation. secured by the respondent from the Office;
(viii) The assessment of damages;
17 June 2003. (ix) Censure; and
(x) Other analogous penalties or sanctions. (Secs. 6, 7, 8, and 9, Executive Order No.
913 [1983]a)

Section 10.2 -
(a) Exercise original jurisdiction in administrative complaints for violations of laws involving
intellectual property rights: Provided, That its jurisdiction is limited to complaints where the


!" DomingoŸAriolaŸMartinezŸMolaer

DAMM NOTES – LAW ON INTELLECTUAL PROPERTY SY $%&! - !"#!
Atty. Castillo-Taleon


shall deceive the public and defraud another of his legitimate trade, or any subsequent
D. Requirement of Notice vendor of such goods or any agent of any vendor engaged in selling such goods with a
like purpose;
The owner of the registered mark shall not be entitled to recover profits or damages unless the (b) Any person who by any artifice, or device, or who employs any other means calculated to
acts have been committed with knowledge that such imitation is likely to cause confusion, or to induce the false belief that such person is offering the services of another who has
Infringement of Trademark Unfair Competition identified such services in the mind of the public; or
Unauthorized use of a trademark Passing off of one’s goods as those of another (c) Any person who shall make any false statement in the course of trade or who shall
commit any other act contrary to good faith of a nature calculated to discredit the goods,
Fraudulent intent is unnecessary Fraudulent intent is essential business or services of another.

Prior registration of the trademark is a Registration is not necessary 168.4. The remedies provided by Sections 156, 157 and 161 shall apply mutatis mutandis. (Sec.
prerequisite to the action 29, R.A. No. 166a)

Limited scope Wider scope

Same class of goods or services must be Different classes of goods or services may be
involved. involved Case No. 1
Pilipinas Shell Petroleum, et al. vs. Romars Int’l (G.R. No. 189669, Feb. 16, 2015)
cause mistake, or to deceive.
FACTS:
Knowledge is presumed if: (a) mark is displayed with the words “Registered Mark” or symbol “®” Petitioner received information that respondent was selling, offering for sale, or distributing LPG
or (b) if the defendant had otherwise actual notice of the registration. by illegally refilling the steel cylinders manufactured by and bearing the duly registered
trademark and device of respondent. After obtaining the services of paralegal investigators, the
petitioners confirmed their suspicion and then requested the NBI to investigate the suspected
activities of the respondent for the purpose of apprehending and prosecuting establishments
conducting illegal refilling, distribution and/or sale of LPG products using the same containers of
Petron and Shell. These acts constitute a violation of Sec. 168 in relation to Sec. 170 of RA 8293
XI. UNFAIR COMPETITION a.k.a. Intellectual Property Code of the Philippines, and/or sec. 2 of RA 623 a.k.a. An Act to
Regualte the Use of Duly Stamped or Marked Bottles, Boxes, Casks, Kegs, Barrels and Other
Section 168. Unfair Competition, Rights, Regulation and Remedies. - Similar Contianers. After finding sufficient proof for violating such laws, NBI filed with RTC Naga
168.1. A person who has identified in the mind of the public the goods he manufactures or deals 2 search warrants for violation of Sec. 155.1, in relation to Sec. 170 pf RA 8293.
in, his business or services from those of others, whether or not a registered mark is employed,
has a property right in the goodwill of the said goods, business or services so identified, which RTC-NAGA
will be protected in the same manner as other property rights. Granted the issuance of such warrants. On the same day, it was served at the respondent’s
premises and seized articles or items described in the warrant.
168.2. Any person who shall employ deception or any other means contrary to good faith by Respondents then filed a motion to quash but it was denied.
which he shall pass off the goods manufactured by him or in which he deals, or his business, or Respondents filed a Motion for Reconsideration and raised for the first time the issue of
services for those of the one having established such goodwill, or who shall commit any acts impropriety of filing the Application for search warrant at RTC-NAGA when it should be in RTC-
calculated to produce said result, shall be guilty of unfair competition, and shall be subject to an IRIGA because the crime was committed within the jurisdiction of the latter.
action therefor. RTC granted Motion for reconsideration.

168.3. In particular, and without in any way limiting the scope of protection against unfair Petitioner appealed to CA but the latter affirmed RTC’s order.
competition, the following shall be deemed guilty of unfair competition:
(a) Any person, who is selling his goods and gives them the general appearance of goods of Hence this petition for review.
another manufacturer or dealer, either as to the goods themselves or in the wrapping of
the packages in which they are contained, or the devices or words thereon, or in any other ISSUE/S:
feature of their appearance, which would be likely to influence purchasers to believe that WON the venue in an application for search warrant is jurisdictional - NO
the goods offered are those of a manufacturer or dealer, other than the actual
manufacturer or dealer, or who otherwise clothes the goods with such appearance as

!" DomingoŸAriolaŸMartinezŸMolaer

DAMM NOTES – LAW ON INTELLECTUAL PROPERTY SY $%&! - !"#!
Atty. Castillo-Taleon


HELD: sensibilities,” or otherwise unlawful. In this case, both characteristics are present. First, both
The application for search warrants is not one involving jurisdiction because the power to issue a parties are competitors or trade rivals, both being engaged in the manufacture of plastic-made
special criminal process is inherent in all courts. automotive parts. Second, the acts of the petitioner were clearly “contrary to good conscience”
as petitioner admitted having employed respondent’s former employees, deliberately copied
respondents products and even went to the extent of selling these products to the respondent’s
Case No. 2 customers. Hence, the petitioner is guilty of unfair competition under Art. 28.
Willaware Products Corp. vs. Jesichris Mfg. Corp. (G.R. No. 195549, Sept. 3, 2014)

FACTS:
Respondent filed a complaint for damages with RTC for unfair competition with prayer for XII. TRADE NAMES OR BUSINESS NAMES
permanent injunction to enjoin the petitioner from manufacturing and distributing plastic-made
automotive parts similar to those of the respondent. Respondent has been manufacturing in its TRADE NAME refers to the name or designation identifying or distinguishing an enterprise; it is
Caloocan plant and distributing plastic-made automotive parts. Petitioner on the other hand is also known as “business identifier”
engaged in the manufacture and distribution of kitchenware items made of plastic and metal.
The respondent later on discovered that the petitioner has been manufacturing and distributing Names That Cannot be Used as a Trade Name
the same automotive parts with exactly similar design, same material and colors but was selling A name or designation may not be used as a trade name if by its nature or the use to which such
these products at a lower price as the respondent’s to the same customers. name or designation may be put, it is contrary to public order or morals and if, in particular, it is
liable to deceive trade circles or the public as to the nature of the enterprise identified by that
RTC ruled in favor of the respondent that the petitioner’s acts of manufacturing similar plastic- name.
made automotive parts and selling the same products to the respondent’s customers, which
cultivated over the years, will have to be enjoined. Hence the petitioner is liable to respondent for
2M pesos as actual damages, 100k as atty’s fees and 100k for exemplary damages. Protected Even Without Registration
165.2.
Petitioner appealed to CA, which affirmed RTC’s ruling with modification. Petitioner asserts that (a) Notwithstanding any laws or regulations providing for any obligation to register trade
if there is no IP protecting a good belonging to another, the copying thereof for production and names, such names shall be protected, even prior to or without registration, against any
selling does not add up to unfair competition as competition is promoted by law to benefit unlawful act committed by third parties.
consumers. Respondent averred that copyright and patent registrations are immaterial for an (b) In particular, any subsequent use of the trade name by a third party, whether as a trade
unfair competition case to prosper under Art. 28 of the Civil Code. It stresses that the name or a mark or collective mark, or any such use of a similar trade name or mark, likely
characteristics of unfair competition are present in the instant case as the parties are trade rivals to mislead the public, shall be deemed unlawful.
and petitioner’s acts are contrary to good conscience for deliberately copying its products and
employing its former employees. 165.3. The remedies provided for in Sections 153 to 156 and Sections 166 and 167 shall apply
mutatis mutandis.
Petitioner moved for reconsideration but it was denied. hence this petition for review.
165.4. Any change in the ownership of a trade name shall be made with the transfer of the
ISSUE/S: enterprise or part thereof identified by that name. The provisions of Subsections 149.2 to 149.4
WON unfair competition under human relations when the parties are not competitors and there shall apply mutatis mutandis.
is actually no damage on the part of the respondent. - YES

HELD:
The instant case falls under Art. 28 of Civil code on human relations and NOT unfair competition XIII. COLLECTIVE MARKS
under RA 8293. The concept of "unfair competition" under Article 28 is very much broader than
that covered by intellectual property laws. Under the present article, which follows the extended COLLECTIVE MARK is also defined by the law as any visible sign designated as such in the
concept of "unfair competition" in American jurisdictions, the term covers even cases of application for registration and capable of distinguishing the origin or any other common
discovery of trade secrets of a competitor, bribery of his employees, misrepresentation of all characteristic, including the quality of goods or services of different enterprises which use the
kinds, interference with the fulfillment of a competitor's contracts, or any malicious interference sign under the control of the registered owner of the collective mark.
with the latter's business. What Art. 28 seeks to prevent is not competition per se but the use of
unjust, oppressive or high-handed methods which may deprive others of a fair chance to engage
in business or to earn a living. In order to qualify the competition as “unfair” it must have 2
characteristics: (1) It must involve an injury to a competitor or trade rival, and (2) it must involve XIV. CASES
acts which are characterized as “contrary to good conscience,” or “shocking to judicial

!" DomingoŸAriolaŸMartinezŸMolaer

DAMM NOTES – LAW ON INTELLECTUAL PROPERTY SY $%&! - !"#!
Atty. Castillo-Taleon


COPYRIGHT
COPYRIGHT

I. DEFINITION OF COPYRIGHT Requisites for the Creation of a Copyrightable Work


1. ORIGINALITY means that the work “owes its origin to the author.”
COPYRIGHT is the right over literary and artistic works which are original intellectual creations
in the literary and artistic domain protected from the moment of creation (Kho v.CA, GR No. What constitutes originality?
115758). a. The work is an independent creation of the author;
b. It must not be copied; and
It is the element of a person’s ownership of his intellectual creation that permits him (author, c. It must involve some intellectual effort.
composer, or artist) to exclusively print, publish, and vend the product of his creation.
• Common law copyright – that which secures to the owner exclusivity until its public 2. EXPRESSION means that there must be “fixation.” To be “fixed,” a work must be
dissemination. embodied in a medium sufficiently permanent or stable to permit it to be perceived,
• Statutory copyright – that which secures protection and exclusivity in the owner by reproduced, or otherwise communicated for a period of more than transitory duration.
force of law even when the work has been made accessible to the public.
II. TERRITORIAL APPLICATION OF COPYRIGHT LAWS (IP CODE)
There is CREATION when an idea is expressed in some tangible medium, or is at least Our copyright laws have no extra-territorial operation and the rights granted under our laws can
expressed in such a way that the critical transition from bare idea or concept to “product” is only be infringed by acts done within our territorial jurisdiction.
effected.
III. COPYRIGHT IS DISTINCT FROM TRADEMARK AND PATENTS
Castillo notes!
Pearl & Dean (Phil), Inc. vs Shoemart, Inc. (2003)
You must remember that the copyright exists from the moment of creation of the work. Pearl & Dean (Phil), Inc. is a corporation engaged in the manufacture of advertising display units
Unlike in patents and unlike in trademarks where there is infringement only if it is registered called light boxes. In January 1981, Pearl & Dean was able to acquire copyrights over the
in the IPO. In copyright from the moment the work is created, the copyright exists. You don’t designs of the display units. In 1988, their trademark application for “Poster Ads” was approved;
even need to register it in the Bureau of copyright of the intellectual property office for they used the same trademark to advertise their light boxes.
copyright to exist. There is only a system of registration. Even the registration itself is not
proof of ownership of the copyright. It is just a prima facie evidence. Something to In 1985, Pearl & Dean negotiated with Shoemart, Inc. (SM) so that the former may be contracted
show that I was first in creating. When you take pictures, the photos, that is your own work to install light boxes in the ad spaces of SM. Eventually, SM rejected Pearl & Dean’s proposal.
and you have a copyright over those photos. If you upload that photo in the internet and
somebody gets it and uses it without your right, there’s an infringement of copyright. Ganon Two years later, Pearl & Dean received report that light boxes, exactly the same as theirs, were
ka lawak ang copyright. being used by SM in their ad spaces. They demanded SM to stop using the light boxes and at
the same time asked for damages amounting to P20 M. SM refused to pay damages though
For example there’s a new building and you take a selfie. Can the guard, upon they removed the light boxes. Pearl & Dean eventually sued SM. SM argued that it did not
authority given to him by the owner of the building, prevent you from taking a photo of that infringe on Pearl & Dean’s trademark because Pearl & Dean’s trademark is only applicable to
building? YES, he can because the building, the design, is owned by the creator. envelopes and stationeries and not to the type of ad spaces owned by SM. SM also averred that
Sometimes there are certain corporations even certain owners of properties like this with “Poster Ads” is a generic term hence it is not subject to trademark registration. SM also averred
original design. Sometimes I don’t want you to take a photo of the building lang itself. Dapat that the actual light boxes are not copyrightable. The RTC ruled in favor of Pearl & Dean. But the
may tao. Court of Appeals ruled in favor of SM.
Ano bang copyright? Anong common doon sa mga subject sa copyright? It is a
literary or artistic work. Sa patent is more of a product or a process. Trademark it’s more of ISSUE: Whether the Court of Appeals is correct
a logo or a device or a name associated or it would not even be an original or a tagline. In
copyright it must be original. And if you notice as I told you earlier it has something to do HELD: YES. The light boxes cannot, by any stretch of the imagination, be considered as either
with a literary or artistic work.? Generally a copyright is a right over artistic and literary works prints, pictorial illustrations, advertising copies, labels, tags or box wraps, to be properly
which are original intellectual property creations and it’s protected form the moment of classified as a copyrightable; what was copyrighted were the technical drawings only, and not
creation. Can you take a photo off the internet and used it in your AVP, the owner of the light boxes themselves. In other cases, it was held that there is no copyright infringement
the photo sued you for copyright infringement, Can you have a defense of good faith when one who, without being authorized, uses a copyrighted architectural plan to construct a
“I didn’t know”? NO. Good faith is not a defense. structure. This is because the copyright does not extend to the structures themselves.


!" DomingoŸAriolaŸMartinezŸMolaer

DAMM NOTES – LAW ON INTELLECTUAL PROPERTY SY $%&! - !"#!
Atty. Castillo-Taleon


On the trademark infringement allegation, the words “Poster Ads” are a simple contraction of the
generic term poster advertising. In the absence of any convincing proof that “Poster Ads” has R filed a motion to quash the search warrants averring that the works covered by the certificates
acquired a secondary meaning in this jurisdiction, Pearl & Dean’s exclusive right to the use of issued by the National Library are not artistic in nature; they are considered automotive spare
“Poster Ads” is limited to what is written in its certificate of registration, namely, stationeries. parts and pertain to technology. They aver that the models are not original, and as such are the
proper subject of a patent, not copyright.

Kho vs. Court of Appeals (2002) RTC quashed the SW. P’s MR having been denied; he filed a petition for certiorari in the CA.
Elidad Kho is the owner of KEC Cosmetics Laboratory and she was also the holder of copyrights The petition was dismissed.
over Chin Chun Su and its Oval Facial Cream Container/Case. She also bought the patent rights
over the Chin Chun Su & Device and Chin Chun Su for medicated cream from one Quintin ISSUES & HELD:
Cheng, who was the assignee of Shun Yi Factory – a Taiwanese factory actually (1) Whether P’s certificate of copyright registration over said utility models are
manufacturing Chin Chun Su products. valid

Kho filed a petition for injunction against Summerville General Merchandising and Company to The petition has no merit. To discharge his burden, the applicant may present the certificate of
enjoin the latter from advertising and selling Chin Chun Su products, in similar containers as that registration covering the work or, in its absence, other evidence. A copyright certificate provides
of Kho, for this is misleading the public and causing Kho to lose income; the petition is also to prima facie evidence of originality which is one element of copyright validity. It constitutes prima
enjoin Summerville from infringing upon Kho’s copyrights. facie evidence of both validity and ownership and the validity of the facts stated in the certificate.
The presumption of validity to a certificate of copyright registration merely orders the burden of
Summerville in their defense alleged that they are the exclusive and authorized importer, re- proof. The applicant should not ordinarily be forced, in the first instance, to prove all the multiple
packer and distributor of Chin Chun Su products; that Shun Yi even authorized Summerville to facts that underline the validity of the copyright unless the respondent, effectively challenging
register its trade name Chin Chun Su Medicated Cream with the Philippine Patent Office; that them, shifts the burden of doing so to the applicant.
Quintin Cheng, from whom Kho acquired her patent rights, had been terminated (her services)
by Shun Yi. A certificate of registration creates no rebuttable presumption of copyright validity where other
evidence in the record casts doubt on the question. In such a case, validity will not be
ISSUE: Whether Kho has the exclusive right to use the trade name and its container presumed. No copyright granted by law can be said to arise in favor of the petitioner despite the
issuance of the certificates of copyright registration and the deposit of the Leaf Spring Eye
HELD: NO. Kho has no right to support her claim for the exclusive use of the subject trade name Bushing and Vehicle Bearing Cushion.
and its container. The name and container of a beauty cream product are proper subjects of a
trademark (not copyright like what she registered for) inasmuch as the same falls squarely (2) Whether P’s utility models can be considered literary and artistic works
within its definition. In order to be entitled to exclusively use the same in the sale of the beauty subject to copyright protection
cream product, the user must sufficiently prove that she registered or used it before anybody
else did. Kho’s copyright and patent registration of the name and container would not guarantee We agree with the contention of the petitioner (citing Section 171.10 of R.A. No. 8293), that the
her the right to the exclusive use of the same for the reason that they are not appropriate author’s intellectual creation, regardless of whether it is a creation with utilitarian functions or
subjects of the said intellectual rights. Consequently, a preliminary injunction order cannot be incorporated in a useful article produced on an industrial scale, is protected by copyright
issued for the reason that the petitioner has not proven that she has a clear right over the said law. However, the law refers to a “work of applied art which is an artistic creation.” It bears
name and container to the exclusion of others, not having proven that she has registered a stressing that there is no copyright protection for works of applied art or industrial design which
trademark thereto or used the same before anyone did. have aesthetic or artistic features that cannot be identified separately from the utilitarian aspects
of the article. Functional components of useful articles, no matter how artistically designed, have
generally been denied copyright protection unless they are separable from the useful article.
Ching vs. Salinas (2005)
P is the owner and general manager of Jeshicris Manufacturing Co., the maker and In this case, the petitioner’s models are not works of applied art, nor artistic works. They are
manufacturer of a Utility Model, described as “Leaf Spring Eye Bushing for Automobile” made up utility models, useful articles, albeit with no artistic design or value. Being plain automotive spare
of plastic, which was issued by the National Library Certificates of Copyright Registration and parts that must conform to the original structural design of the components they seek to replace,
Deposit. the Leaf Spring Eye Bushing and Vehicle Bearing Cushion are not ornamental. They lack the
decorative quality or value that must characterize authentic works of applied art. They are not
P requested the NBI for police/investigative assistance for the apprehension and prosecution of even artistic creations with incidental utilitarian functions or works incorporated in a useful article.
illegal manufacturers, producers and/or distributors of the works. After due investigation, the NBI In actuality, the personal properties described in the search warrants are mechanical works, the
filed applications for SWs against R alleging that the latter therein reproduced and distributed the principal function of which is utility sans any aesthetic embellishment.
said models penalized under R.A. No. 8293. RTC granted the application and issued SWs for
the seizure of the aforecited articles.

!" DomingoŸAriolaŸMartinezŸMolaer

DAMM NOTES – LAW ON INTELLECTUAL PROPERTY SY $%&! - !"#!
Atty. Castillo-Taleon


In this case, the bushing and cushion are not works of art. They are, as the petitioner himself
admitted, utility models which may be the subject of a patent. IV. COPYRIGHT OVER LITERARY AND WORKS IS VESTED FROM THE MOMENT
OF CREATION
IN LIGHT OF ALL THE FOREGOING, the instant petition is hereby DENIED for lack of merit.
The assailed Decision and Resolution of the CA are AFFIRMED. • Literary and artistic works are original intellectual creations in the literary and artistic
domain protected from the moment of their creation (IPC, Section 171.1).
• DENICOLA TEST inquires into which aspects of the work are dictated by the • Works are protected by the sole fact of their creation, irrespective of their mode or
functional constraints of the article and which aspects reflect unconstrained form of expression, as well as of their content, quality and purpose (IPC, Section
perspective of the artist (Prof. Robert Denicola). 171.2).
• STATUTORY COPYRIGHT is conferred by the statute when the work is made under
Castillo notes! the Intellectual Property Code. In other words, publication is not required. Copyright
subsists from the moment of creation. Acquisition of copyright is not contingent or
In 2009, in the bar, there was a question about the DENICOLA DOCTRINE. Yung dependent on any formality or registration (Unilever Phils., Inc. vs. CA, 498 SCRA
DENICOLA is the family name of a particular professor in intellectual property. There is an 334).
issue whereby you have a design but the design for an article is more of a combination of a
utility pero artisitic siya. If your ornamental design is coupled with the need for a utilitarian
need, is that copyrightable? Merong dito combination of utility and artistic. Sabi ni Prof. V. WORKS PROTECTED BY COPYRIGHT
Denicola, subject pa ng copyright yan kapag yung design mo mas nangangailan yung utility
sa artistic. If you could separate the artistic part from the utilitarian part, that artistic part is Original Works (IPC, Section 172.1):
copyrightable. Pero kapag hindi mo siya maihiwalay, yung artistic part sa utitlitarian purpose 1. Books, pamphlets, articles and other writings;
ng design, then that is NOT copyrightable. Parang yung design ng cup. Kunwari yung • It matters not whether the works are published or not, whether they be in verbal or
design mo ng cup may kung ano-ano pang design na artistic na kahit tangglain mo yon numerical symbol.
magagamit parin yung cup yung part nay un, subject to copyright. Pero pag tinanggal mo at
nawala yung gamit ng cup, kasi they are so intertwined with one another, that is not subject 2. Periodicals and newspapers;
to copyright. That’s actually the Denicola Test. I do not expect that because it has been • Under IPC, Section 175, “news of the day and other miscellaneous facts having
asked in the past uh but there might be a question of whether or not a design which is the character of mere items of press information” are unprotected subject matter.
mutually required for it’s utility is subject to copyright. Ornamental okay? Ornamental dapat The facts that constitute the news are unprotected, but the copyright protection
yung design. subsists in any work entailing intellectual effort that may result in the creation of a
news report.

3. Lectures, sermons, addresses, dissertations prepared for oral
delivery, whether or not reduced in writing or other material form;
• While a speech or lecture is being mulled over by its author mentally, there is, as
yet nothing to protect. The moment, however, it is delivered—whether or not from
a written text or from memory—there is protected material (IPC, Section 188.3).

4. Letters;
• Letters and other private communications in writing are owned by the person to
whom they are addressed or delivered, but they cannot be published or
disseminated without the consent of the writer or his heirs (CIVIL CODE, Article
723).

NOT 5. Dramatic or dramatico-musical compositions; choreographic works or


COPYRIGHTABLE entertainment in dumb shows;
COPYRIGHTABLE
6. Musical compositions, with or without words;
• Can an article of commerce serve as a trademark and at the same time enjoy
patent and copyright protection? 7. Works of drawing, painting, architecture, sculpture, engraving,
lithography or other works of art; models or designs for works of art;

!" DomingoŸAriolaŸMartinezŸMolaer

DAMM NOTES – LAW ON INTELLECTUAL PROPERTY SY $%&! - !"#!
Atty. Castillo-Taleon


Requirements of originality – an original work is that which requires originality in skill or labor
8. Original ornamental designs or models for articles of manufacture, in execution such that the works became individual either in matter, forms, arrangement or
whether or not registrable as an industrial design, and other works of treatment (not necessarily original thought, idea, or research)
applied art;

• Work of Applied Art – an artistic creation with utilitarian functions or incorporated in Sambar vs Levi Strauss (2002)
a useful article, whether made by hand or produced on an industrial scale (IPC, Private respondents alleged in their complaint that Levi Strauss and Co. (LS&Co.), an
Sections 171.10). internationally known clothing manufacturer, own the arcuate design trademark which was
registered under US Trademark Registration No. 404,248 on November 16, 1943. That
9. Illustrations, maps, plans, sketches, charts and three-dimensional sometime in 1987, CVSGIC and Venancio Sambar, without the consent and authority of private
works relative to geography, topography, architecture or science; respondents and in infringement and unfair competition, sold and advertised, and despite
• What is copyrightable in a map is the selection, arrangement, and presentation of demands to cease and desist, continued to manufacture, sell and advertise denim, pants under
the component parts. the brand name “Europress” with back pockets bearing a design similar to the arcuate trademark
of private respondents, thereby causing confusion on the buying public, prejudiced to private
10. Drawings or plastic works of a scientific or technical character; respondent’s goodwill and property right.

11. Photographic works including works produced by a process Sambar filed a separate answer. He admitted that copyright Registration No. 1-1998 was issued
analogous to photography; lantern slides; to him, but he denied using it. He said he did not authorize anyone to use the copyrighted
design.
12. Audiovisual works and cinematographic works and works produced
by a process analogous to cinematography or any process for making Trial court issued a writ of preliminary injunction enjoining CVSGIC and petitioner from
audio-visual recordings; manufacturing, advertising and selling pants with the arcuate design on their back pockets.

13. Pictorial illustrations and advertisements; Private respondents moved for reconsideration praying for the cancellation of petitioner’s
copyright registration. Trial court granted the prayer. Petitioner appealed to the Court of Appeals
14. Computer programs; and which affirmed the ruling of the trial court.
• Computer programs – set of instructions expressed in words, codes, schemes or
in any other form, which is capable, if incorporated in a medium that the computer ISSUE: Whether petitioner infringed on private respondent’s arcuate design
can read, of causing the computer to perform or achieve a particular task or result
(IPC, Section 171.4). HELD: YES. To be entitled to a copyright, the thing being copyrighted must be original, created
by the author through his own skill, labor and judgment, without directly copying or evasively
15. Other literary, scholarly, scientific and artistic works. imitating the work of another. Both the trial court and the Court of Appeals found there was
infringement.

Derivative Works (IPC, Section 173):


These are works based upon one or more preexisting works, such as translation, musical
arrangement, dramatization, fictionalization, motion picture version, sound recordings, art
reproduction, abridgement, condensation or any other form in which a work may be recast,
transformed or adapted (17 USC, Section 101).
It includes:
1. Dramatizations, translations, adaptations, abridgments, arrangements, and
other alterations of literary or artistic works; and
2. Collections of literary, scholarly or artistic works, and compilations of data
and other materials which are original by reason of the selection or
coordination or arrangement of their contents.


!" DomingoŸAriolaŸMartinezŸMolaer

DAMM NOTES – LAW ON INTELLECTUAL PROPERTY SY $%&! - !"#!
Atty. Castillo-Taleon


Castillo notes!

So in order to have your copyright over a material object or your intellectual creation, you Castillo notes!
must take note of the 2 ELEMENTS OF COPYRIGHTABILITY: (1) ORIGINALITY – the
work is created by the author’s own skill. 2 elements of this are (a) the work is created by What is copyrightable is the finished product. We take not also that the new of
the author’s independent work and (b) That the work must possess a degree of creativity; the day or mere items of information, hindi rin ito copyrightable. Official text of legislative,
And (2) EXPRESSION – that work must be expressed or embodied in a particular medium administrative or legal nature kaya pag gumagawa ka ng pleading, nacicite mo yon. Yung
which is more or less permanent. Ang tawag diyan, pag naririnig niyo sa mga media, there decisions of the court of the supreme court. Pano yung SCRA or CD Asia, they also contain
must be “Fixation”. Ibig sabihin niyon yung work mo is embodied in a particular medium decisions of the SC, are they subject to copyright? YES, kasi yung products nila may quick
kaya ka may CD, mayrong Cassette tape, laser disc yung mga movie. That’s the reason index yung sinasabi ng law na not subject to copyright is yung original decisions. Yung scra
why you take note of those works which are not copyrightable. Ano ano yung mga hindi pag nakikita mo, may mga ID yung arrangement mismo that’s all subject to copyright coz
copyrightable? Yung concept, there was a case, yung dating game. Hindi mo pwedeng that’s original eh. And then the work of government. Any work of the government is not
sabihing CONCEPT KO YAN EH! Yung merong searchee merong searcher. Makikita mo subject to copyright. You take note that prior approval of the government agency or office
yung showtime at eat bulaga, minsan pare-pareho sila ng games because the concept is wherein the work is created shall be necessary for exploitation of such work for profit. Such
not subject of a copyright. That concept must be embodied in a fixation. So what is agency or office may among other thigs impose the payment of royalties. Ang DOST they
copyritable is yung show mismo. Okay? The eat bulaga mismo or that particular game or have a lot of creations. If you use them and you want to exploit it and for profit. You need a
the movie itself. Kaya minsan pare-pareho yung tema ng movie or TV show. But the movie prior approval because you need to pay royalties. Pero kapag statutes lang, for purposes
where you express your concept, your idea that is the one that’s copyrightable. So you take statutes, rules and regulations, sermons, lectures, your speeches, hindi kailangan ng prior
note lang yung mga non-copyrightable for purposes of the bar and even for your final exams approval. Pero pag for profit, kailangan.
Sec. 175 anong hindi copyrightable? Concept, Idea, procedure, system, method, operations
discovery, and mere data.
VII. RIGHTS CONFERRED BY COPYRIGHT
VI. WORKS NOT PROTECTED BY COPYRIGHT 1. Economic rights (IPC, Section 177)
Exclusive right to carry out, authorize or prevent the following acts:
Works Not Protected: (1) Reproduction of the work or substantial portion of the work;
1. Pleadings; Reproduction – the making of one or more copies, temporary or permanent, in whole or in part,
2. Idea, procedure, system method or operation, concept, principle, discovery or mere of a work or a sound recording in any manner or form without prejudice to the provisions of
data as such, even if they are expressed, explained, illustrated or embodied in a work Section 85
(IPC, Section 175); (2) Dramatization, translation, adaptation, abridgment, arrangement or other
• Format or mechanisms of a television show are not copyrightable. The law in transformation of the work;
enumerating what are subject to copyright refers to finished works and are not to (3) First public distribution of the original and each copy of the work by sale or
concepts (Joaquin, Jr. vs. Drilon, GR No. 108946); other forms of transfer of ownership;
3. News of the day and other miscellaneous facts having the character of mere items of (4) Rental of the original or a copy of an audiovisual or cinematographic work;
press information (IPC, Section 175); Rental – the transfer of the possession of the original or a copy of a work for a limited period of
4. Decisions of courts and tribunals; time, for profit0making purposes
• This pertains to the “original decisions,” not the SCRA-published volumes since (5) Public display of the original or a copy of the work;
these are protected under derivative works under Section 173.1(b). (6) Public performance of the work; and
5. Any official text of a legislative, administrative or legal nature, as well as any official (7) Other communication to the public of the work.
translation thereof (IPC, Section 175); and
6. Any work of the Government of the Philippines. Economic rights allow the owner to derive financial reward from the use of his works by others.

General Rule: Prior approval of the government agency or office wherein the work is created
shall be necessary for exploitation of such work for profit. Such agency or office may, among
other things, impose as a condition the payment of royalties.
• Exception:
No prior approval or conditions shall be required for the use of any purpose of statutes, rules and
regulations, and speeches, lectures, sermons, addresses, and dissertations, pronounced, read
or rendered in courts of justice, before administrative agencies, in deliberative assemblies and in
meetings of public character.


!" DomingoŸAriolaŸMartinezŸMolaer

DAMM NOTES – LAW ON INTELLECTUAL PROPERTY SY $%&! - !"#!
Atty. Castillo-Taleon


Castillo notes! (4) To restrain the use of his name with respect to any work not of his own creation or in a
distorted version of his work.
Is plagiarism is the same as copyright infringement? You take note that in copyright there
are 2 rights there (1) Economic right – Sec. 177, 193, 2000(?), and (2) is your Moral Right. These rights are distinct from economic rights and remain with the author even after he has
In General your Economic Rights is your exclusive right to carry out, authorize or prevent transferred or assigned to another “other rights of copyright.”
the reproduction of your work or substantial portion. The dramatization, translation,
adaptation or arrangement or transformation of your work. You have also the exclusive right Moral rights allow the author to take certain actions to preserve the personal link between
to carry out, authorize or prevent the first public distribution of your work or the rental of himself and the work.
your work or the public display of your work or the public performance of your work. Ikaw
lang lahat yun. Your copyright says if you give a right for public performance, that right only
extends to the first public performance it doesn’t include, unless it is in the agreement, yung Castillo notes!
subsequent yung mga replace. For example yung mga artista na yan may mga singers,
kumikita yan ng malalaki kasi nag appear siya sa ASAP. Yung copyright niya lang with Ang Moral Right, these are what you call under Sec. 193, Moral rights are personal rights
respect to that, pag nireplay yun kailangan may bayad din siya, unless of course part yon ng independent from economic rights. So ano tong mga Moral rights na yon, require that the
kanyang contract na the TV station has the right to replay or to use yung kanyang songs or ownership of the rights be attributed to him with the right to attribution, the right to make
yung kanyang performance in TV. Every performance is subject to a royalty. More or less alterations of his work or to prevent the alterations of his work, or to prevent or withhold the
that is the Economic right. publication of his work; the right to preserve the integrity of his work. Yung ayaw nila
gumawa ng mga distorted modifications with regard to their work. Kaya there was a time
when there was a particular director, there was a pinoy movie wherein the actor introduced
something to his work. So there were certain scenes that were directed by the actor and
Castillo notes! then the Director said, that’s not my work any more because you have, ewan ko kung si
Robin Padilla yun, in otherwords, pinakialaman nila yung trabaho ng director. Bida sila
Can corporation/partnership/any juridical entity own or possess economic right? Can Producers sila but they have shot some scenes which they wanted and there was this
they have copyrights? YESSSS. Kaya nga sinasabi ko kanina even a recording company, director who said that’s no longer my work and demanded that he wouldn’t be put as the
yung mga producers ng ABSCBN, they won a lot of copyrights yung mga yan. But do they director of the work because that’s his right that is moral. The right to restrain the use of
have economic rights? YES!!! Do they have moral rights? NOOOOOO. Yung moral rights is his______ with respect to any work not of his own creation. kasi minsan sinasabi, gamitin
given only to a natural person. So that when the creator assigned the work and his copyright mo nalang pangalan mo, but that’s not my work anymore! So he has the right, the moral
to a company or a juridical entity, the juridical entity acquires the copyright inclusive of right. Yun yung economic vs. moral right.
economic rights but not the moral rights kasi siya parin yun. So that if you violate the
economic rights of the economic right holder, there is copyright infringement. If you will
violate, particularly a moral right of a person that is attributing the right to have the work
attributed to you, you use his work without attributing to him, there is plagiarism. So nakita 3. Right to participate in the gross proceeds of the sale or lease of the original
work or Driot de suite (IPC, Section 200); and
niyo na yung difference ng copyright and plagiarism? Yun na more or less plagiarism is
using the work or a particular paragraph of a particular work without attributing the DROIT DE SUITE is the inalienable right to receive to the extent of 5% of the gross proceeds of
ownership or acknowledging the creator. More or less a violation of the right to attribution the sale or lease of a work (IPC, Section 200).
which is a moral right.
Requisites:
a. Sale or lease;
2. Moral rights (IPC, Section 193); b. Of an original work;
(1) Right of Paternity – To require that the authorship of the works be attributed to him, in c. Of painting or sculpture, or of the original manuscript of a writer or composer; and
particular, the right that his name, as far as practicable, be indicated in a prominent d. Subsequent to the first disposition thereof by the author.
way on the copies, and in connection with the public use of his work;
(2) To make any alterations of his work prior to, or to withhold it from publication;
(3) Right of Integrity – To object to any distortion, mutilation or other modification of, or
other derogatory action in relation to, his work which would be prejudicial to his honor
or reputation; and


!" DomingoŸAriolaŸMartinezŸMolaer

DAMM NOTES – LAW ON INTELLECTUAL PROPERTY SY $%&! - !"#!
Atty. Castillo-Taleon


4. Rights of performers, producers of sounds recording and broadcasting General Limitations (IPC, Section 184):
organizations (IPC, Sections 203, 204, 206, 208). 1. Recitation or performance of a work, once it has been lawfully made accessible to
the public, if done privately and free of charge or if made strictly for a charitable or
religious institution or society;
2. The making of quotations from a published work if they are compatible with fair use
Castillo notes! and only to the extent justified for the purpose, including quotations from newspaper
articles and periodicals in the form of press summaries: Provided, That the source
What is the concept of Droit De Suite? The English meaning of that is that that is the right and the name of the author, if appearing on the work, are mentioned; (Sec. 11,
to proceed in subsequent transfers or follow up. There are what you call Follow – up rights. Third Par., P. D. No. 49)
Which means, your copyright owner rights to subsequent transfers. Under the general 3. Reproduction or communication to the public by mass media of articles on current
concept of Droit De suite the _______ has an inalienable right to receive to the extent of 5% political, social, economic, scientific or religious topic, lectures, addresses and other
of the gross proceeds of the sale or release of a work of painting, sculpture or manuscript of works of the same nature, which are delivered in public if such use is for information
purposes and has not been expressly reserved: Provided, That the source is clearly
a writer, composer subsequent to its first disposition. Kaya yung mga ano diyan yung mga
indicated;
paulit like yung mga creators ni Darna. Ni Mars Ravelo. Hanggang ngayon diba? It’s coz 4. Reproduction and communication to the public of literary, scientific or artistic works
they have this Droit De Suite. Ngayon kahit patay na siya (Creator), yung anak niya meron as part of reports of current events by means of photography, cinematography or
parin because they have the right, the heirs have that right 50 years after the death of the broadcasting to the extent necessary for the purpose;
creator. 5. Inclusion of a work in a publication, broadcast, or other communication to the
public, sound recording or film, if such inclusion is made by way of illustration for
Assuming that you have entered into a contract showing that you are a writer and for you to teaching purposes and is compatible with fair use: Provided, That the source and of
the name of the author, if appearing in the work, are mentioned;
write a book. And then you finish your book and sabi ng publishing company, The book of
6. The recording made in schools, universities, or educational institutions of a work
Mr. XYZ will be out in the market by Nov. 30. Tapos biglang nag bago isip mo, “I don’t like to included in a broadcast for the use of such schools, universities or educational
publish my book”. Do you have that right? YES. That is one moral right, to withhold the institutions: Provided, That such recording must be deleted within a reasonable
publication. Can the publishing company file a case to mandate you by a petition for period after they were first broadcast: Provided, further, That such recording may
mandamus to compel you to publish that book because you are required under your not be made from audiovisual works which are part of the general cinema repertoire
contract.kasi diba pwede kang specific performance. Ano sabi sa obligations and contracts, of feature films except for brief excerpts of the work;
pag work? you cannot be held liable. Pag ayaw mo, you will be liable for damages under art. 7. The making of ephemeral recordings by a broadcasting organization by means of
its own facilities and for use in its own broadcast;
1170 (Contravention of Tenor) of the NCC.
8. The use made of a work by or under the direction or control of the Government, by
the National Library or by educational, scientific or professional institutions where
such use is in the public interest and is compatible with fair use;
9. The public performance or the communication to the public of a work, in a place
VIII. LIMITATIONS OF COPYRIGHT where no admission fee is charged in respect of such public performance or
communication, by a club or institution for charitable or educational purpose only,
Limitations to the Rights of Copyright: whose aim is not profit making, subject to such other limitations as may be provided
1. General limitations (IPC, Section 184); in the Regulations; (n)
2. Fair use (IPC, Section 185); 10. Public display of the original or a copy of the work not made by means of a film,
3. Private reproduction of published work in a single copy by a natural person for slide, television image or otherwise on screen or by means of any other device or
research and private study (IPC, Section 187); process: Provided, That either the work has been published, or, that original or the
4. Reprograpihic reproduction in a single copy by non-profit libraries, under certain copy displayed has been sold, given away or otherwise transferred to another
circumstances (IPC, Section 188); person by the author or his successor in title; and
5. Reproduction, under certain circumstances, of a computer program in one back-up 11. Any use made of a work for the purpose of any judicial proceedings or for the giving
copy by the lawful owner of the program (IPC, Section 189); and of professional advice by a legal practitioner
6. Importation for personal purposes under certain conditions (IPC, Section 190).
Fair use of Copyright Work – Section 185
Reproduction of Computer Program – Section 189
Importation for Personal Purposes – Section 190


!" DomingoŸAriolaŸMartinezŸMolaer

DAMM NOTES – LAW ON INTELLECTUAL PROPERTY SY $%&! - !"#!
Atty. Castillo-Taleon


IX. OWNERSHIP OF COPYRIGHT However, the court may authorize their publication or dissemination if
the public good or interest of justice so requires.
CREATOR TO WHOM IT BELONGS

Single Creator Author of the work, his heirs, or assigns


X. DURATION OF COPYRIGHT

If work consists of unidentifiable parts: co-authors jointly as co- TYPE OF WORK DURATION
owners, unless there is agreement to the contrary
Single Creation Lifetime of the creator and in perpetuity after his death
Joint Creator
If the work consists of identifiable parts: author of each part owns
the part that he has created Lifetime of the last surviving co-creator and in
Joint Creation
perpetuity after his death

If the creation is part of his regular duties: employer, unless there is


agreement to the contrary 50 years after the date of their first publication;
Employee’s Creation
except where, before the expiration of the said period,
Anonymous or pseudonymous work
If it is not: employee the author’s identity is revealed or is no longer in doubt,
the first two mentioned rules shall apply; or if
unpublished, 50 years from their making.
Work itself: The person who commissioned the work

Commissioned Work Work of applied art, an artistic 25 years from the date of making
Copyright: Creator, unless there is a written stipulation to the creation with utilitarian functions or
contrary incorporated in a useful article,
whether made by hand or produced
on an industrial scale
For exhibition purposes: Producer
Cinematographic Photographic work, audiovisual work
50 years from the publication of the work, or if
Works produced by photography or
For all other purposes: Producer, author of the scenario, composer, unpublished, 50 years from making the same
analogous processes
film director, author of the work

Newspaper article Lifetime of the author and in perpetuity after his death
Publishers are deemed representative of the author, unless:
1. The contrary appears;
Anonymous and
2. Pseudonyms or adopted name leaves no doubt as to the
pseudonym works
author’s identity;
3. Author discloses his identity

Contributor is deemed to have waived his right, unless he expressly


Collective Works
reserves it

Letters
Writer owns the copyright, but the recipient owns the letter itself.

!! DomingoŸAriolaŸMartinezŸMolaer

DAMM NOTES – LAW ON INTELLECTUAL PROPERTY SY $%&! - !"#!
Atty. Castillo-Taleon


XI. INFRINGEMENT COPYRIGHT QUIZ
INFRINGEMENT or piracy is any violation of the owner’s exclusive rights conferred by law, while
PLAGIARISM is confined to the incorporation in one’s own work that of another without the 1. Ornamental designs or models for articles of manufacture may be registered as an
proper acknowledgement thereof.
industrial design but not copyright.
• Animus furandi or intention to pirate is not an element of copyright infringement.
• An acknowledgement of the sources of the work infringed is not a defense to an FALSE. Sec. 172 of the IPC provides that "original ornamental designs or models for articles of
infringement action. manufacture, whether or not registrable as an industrial design, and other works of applied
Sections 216, 217 art" are protected by the sole fact of their creation.
Presumption of Authorship – Section 219
2. The creation of derivative works such as dramatizations, adaptations, or arrangements
of previously created original works necessarily secures for the creator of the
XII. PRESCRIPTION OF ACTION derivative work a copyright in such original works.
Section 226
FALSE. Sec. 173.2 of the IPC provides, "These works shall be protected as new works,
provided, however, that such new work shall not affect the force of any subsisting copyright
upon the original works employed or any part thereof, or be construed to imply any right to
such use of the original works, or to secure or extend copyright in such original works."

3. The government is absolutely precluded to own copyright.



FALSE. Sec 176.3 provides that "the Government is not precluded from receiving and holding
copyrights transferred to it by assignment, bequest, or otherwise..."

4. In all instances of work created by the author during and in the course of his
employment, the copyright shall belong to the employer provided that the work is the
result of the performance of his regularly assigned duties.

FALSE. Sec. 178.3(b) provides that "the copyright shall belong to the employer, if the work is
the result of the performance of his regularly-assigned duties, UNLESS THERE IS AN
AGREEMENT, express or implied, TO THE CONTRARY."

5. A joint owner of a literary, photographic or artistic work may not grant a license
without the prior written consent of the other joint owner/s.

TRUE. Sec. 180.3 provides that "if two (2) or more persons jointly own a copyright or any part
thereof, neither of the owners shall be entitled to grant licenses without the prior written
consent of the other owner or owners."


!" DomingoŸAriolaŸMartinezŸMolaer

DAMM NOTES – LAW ON INTELLECTUAL PROPERTY SY $%&! - !"#!
Atty. Castillo-Taleon


6. The transfer or assignment of a copyright does not itself constitute a transfer of the cafeteria operator said he purchased the painting from Bernie who represented
material object but the transfer of the material object itself necessarily includes the himself as its painter and owner. Rudy and the cafeteria operator immediately
transfer of the copyright over it. confronted Bernie. While admitting that he did not do the painting, Bernie claimed
ownership of its copyright since he had already registered it in his name with the
National Library as provided in the Intellectual Property Code. Who owns the
FALSE. Sec. 181 provides that "the transfer or assignment of the copyright shall not itself copyright of the painting? Explain.
constitute a transfer of the material object. Nor shall a transfer or assignment of the sole
copy or of one or several copies of the work imply transfer or assignment of the copyright." Rudy owns the copyright to the painting. The registration and deposit of work is purely for
recording the date of registration and deposit of the work shall not be conclusive as to
7. Memes are subject to copyright. copyright ownership or the term of the copyrights or the rights of the copyright owner,
including neighboring rights. In addition, the work need not be registered to be protected.
TRUE. Provided it has all the elements of copyrightability (i.e., originality, etc.). The law provides that intellectual creations in the literary and artistic domain are protected
from the moment of its creation.
8. The publication of the photograph in the Philippine Daily Inquirer of the latest
painting of a national artist as part of a news report of current events does not 12. While vacationing in Boracay, Valentino surreptitiously took photographs of his
constitute infringement of copyright. girlfriend Monaliza in her skimpy bikini. Two weeks later, her photograph appeared in
the internet an in a national celebrity magazine.
TRUE. Sec. 12 of P.D. 49 states that “reproduction and communication to the public of Monaliza found out that Valentino had sold the photographs to the magazine, adding
literary, scientific, or artistic works as part of reports of current events shall not constitute insult to injury, uploaded them to his personal blog on the internet.
copyright infringement.”
Can Monaliza file a complaint against Valentino for damages based on, among other
9. The copyright in a work of architecture shall include the right to control the grounds, violations of her intellectual property right? Does she have any cause of
reconstruction or rehabilitation of the same style as the original of a building to which
action?
the copyright relates.

Monaliza cannot file a complaint against Valentino for damages based on violations of her
FALSE. Sec. 186 provides that "the copyright in any such work shall not include the right to
intellectual property right. However, Monaliza has a cause of action under the Civil Code for
control the reconstruction or rehabilitation in the same style as the original of a building to
violation of privacy/abuse of right.
which the copyright relates."

10. The private reproduction of an entire book in a single copy if made exclusively for
private study does not constitute copyright infringement.

FALSE. Sec. 187.2 provides that "the permission shall not extend to the reproduction of... an
entire book, or a substantial part thereof, or of a musical work in which form by reprographic
means."

11. Rudy is a fine arts student in a university. He stays in a boarding house with Bernie as
his roommate. During his free time, Rudy would paint and leave his finished works
lying around the boarding house. One day, Rudy saw one of his works—an abstract
painting entitled Manila Traffic Jam—on display at the university cafeteria. The

!" DomingoŸAriolaŸMartinezŸMolaer

Potrebbero piacerti anche